Pretest Pediatrics

¡Supera tus tareas y exámenes ahora con Quizwiz!

A 16-year-old boy in Alaska has a syncopal episode while repairing his family's fishing boat. He is found to have glossitis on exam, and a macrocytic anemia with thrombocytopenia. Treatment?

*Diphyllobothrium latum* is the longest tapeworm affecting humans. It is frequently asymptomatic, but can present with megaloblastic anemia as the worm will use vitamin B12 for its own growth. Praziquantel is currently the treatment of choice.

Weakness in what structures leads to an indirect inguinal hernia and direct inguinal hernia. How to manage?

A congenital indirect inguinal hernia is the result of incomplete closure of the processus vaginalis. Direct inguinal hernia, caused by weakness in the musculature of the inguinal canal Consult surgery.

What disease is associated with anti-Saccharomyces cerevisiae antibodies (ASCA)? What disease is associated with anti-neutrophil cytoplasm antibodies (p-ANCA)?

ASCA = Crohn's Disease p-ANCA = Ulcerative Colitis

Top 3 causes of Cushing syndrome

Administration of exogenous adrenocorticotropic hormone or of glucocorticoids is the most common cause of Cushing syndrome. It can also be caused by bilateral adrenal hyperplasia. If the patient were an infant, the most likely answer would be an adrenal carcinoma.

What are the causes of SJS?

Among the known causes of the Stevens-Johnson syndrome are allergy to various drugs (including phenytoin, barbiturates, sulfonamides, and penicillin) and infection with a variety of organisms including Mycoplasma pneumoniae or herpes type 1. Erythema multiforme is sometimes confused with urticaria early in the course because both can have a target-like lesion.

Treatment of Maples Syrup Urine Disease?

Symptoms begin several days after birth and rapidly progress to convulsions and death in 2 to 4 weeks if not treated. The distinctive odor of caramel or maple syrup starts after 1 to 2 days of life, but is variable in intensity. Avoid intake of leucine, isoleucine, and valine.

How to manage a natal tooth?

Anx unstable natal tooth may fall out and be aspirated by the infant. Extraction by a pediatric dentist is the usual course when the tooth is loose; however, extraction can lead to abnormal spacing of the remaining teeth as they erupt.

Describe phenylketonuria.

Autosomal recessive pattern of inheritance, is caused by the absence of an enzyme that metabolizes phenylalanine to tyrosine. Treatment consists of a diet that maintains phenylalanine at levels low enough to prevent brain damage but adequate to support normal physical and mental development.

What is biotinidase deficiency?

Biotinidase is the enzyme responsible for breakdown of biocytin (the lysyl precursor of biotin) to free biotin. Deficiency of the enzyme, which is inherited as an autosomal recessive trait, results in malfunctioning of the biotin-dependent mitochondrial enzymes and in organic acidemia. Clinical problems related to the deficiency appear several months or years after birth and include dermatitis, alopecia, ataxia, hypotonia, seizures, developmental delay, deafness, immunodeficiency, and metabolic acidosis. The treatment is lifelong administration of free biotin.

What is blood in the anterior chamber of the eye called?

Blood in the anterior chamber of the eye is called a hyphema.

Treating an undescended teste eliminates the risk of which of the following? a. Testicular malignancy b. Decreased sperm count c. Torsion of testes d. UTI e. Epididymitis

C. Torsion of the testes is practically eliminated because bringing the teste down includes orchiopexy. low sperm count and malignancy will still be increased in these patients compared to the normal population even after the orchiopexy.

What is the most commonly fractured bone in newborns?

Clavicle. If there is a mass on the clavicle then you are probably good to just reassure the parents. Clavicular fracture may happen in any delivery, although there is higher risk with large-for- gestational-age infants.

When to surgically close cleft lip? Cleft palate?

Cleft lip: within the first 2-3 months of life. Cleft palate: 6 months - 5 years.

479. A 15-year-old girl is seen in your clinic with a sprained ankle, which occurred the previous day while she was exercising in her room. You realize that you have not seen her for quite some time, and begin to expand your examination beyond the ankle. You find relatively minimal swelling on her right ankle. She has dental decay, especially of anterior teeth and a swollen, reddened, irritated uvula. She seems to be somewhat hirsute on her arms and legs, but has thinning of her hair of the head. She has a resting heart rate of 60 beats per minute, and her oral temperature is 35.5°C (96°F). Further questioning suggests that she has developed secondary amenorrhea. Which of the following is the most appropriate next step in the management of this girl? What is her diagnosis?

Comparison of current and past weights Dx: Bulemia

What is congenital cystic adenomatoid malformation?

Congenital cystic adenomatoid malformation (CCAM) is thought to arise from an embryonic disruption before the 35th day that causes improper development of bronchioles. The cystic mass is usually identified on prenatal ultrasound around the 20th week. Large lesions (as that noted on the radiograph on the patient's right side) may compress the affected lung and cause pulmonary hypoplasia, and cause midline shift away from the lesion (note the heart is shifted toward the patient's left on the radiograph). Treatment is typically surgical excision of the affected lobe. Some patients may be at risk for primary pulmonary malignancy.

95. A 3-day-old infant was born to a mother with active systemic lupus erythematosus (SLE) Name the cardiac abnormality

Congenital heart block.

311. On a newborn boy's first examination, you note a prominent occiput, a broad forehead, and an absent anterior fontanelle. The baby's head is long and narrow. The remainder of the physical examination, including a careful neurological evaluation, is normal. You note that the baby was born via cesarean section for cephalopelvic disproportion. When you enter the mother's room, the first question she asks is about her baby's head shape. What is the diagnosis? Treatment?

Craniosynostosis This condition is usually sporadic (occurring in 1 in 2000 births) and more commonly affects the sagittal suture, which results in scaphocephaly (a long and narrow skull). In general, premature fusion of a single suture does not cause increased intracranial pressure or hydrocephalus; these features are more common with premature fusion of two or more sutures. The therapy for this condition is controversial, but usually involves surgery; consultation with a neurosurgeon would be indicated.

An infant born at home 2 days ago who has profuse purulent drainage from both eyes, lid edema, and chemosis. What diagnostic test should be ordered?

Culture on modified Thayer-Martin medium

A 2-year-old boy has 11 days of nonbloody diarrhea, accompanied by fatigue, a distended abdomen with cramps, fever, and weight loss. The mother notes that the day care got some "bad raspberries" and many children have the same symptoms. Treatment?

Cyclospora cayetanensis can cause prolonged diarrhea after an incubation period of about 7 days. Exposure is through contaminated foods. Diagnosis is confirmed by identifying oocysts in the stool. Trimethoprim/sulfamethoxazole is the treatment of choice; ciprofloxacin is an alternative.

22. You are performing a well-child examination on the 1-year-old child shown in the picture. For this particular problem, which of the following is the most appropriate next step in management?

e. Refer immediately to ophthalmology

A 15-year-old defensive lineman for his high school football team whose mother reports that his shoulder pads have permanently stained his neck. What is the diagnosis?

g. Type 2 diabetes

61. Irritability, convulsions, hypochromic anemia

g. Vitamin B6 deficiency

271. A 3-day-old infant born at home is brought to the emergency center with bloody stools, hematemesis, and purpura. His circumcision is oozing blood.

g. Vitamin K

A 4-year-old girl who spends 3 days a week in Mother's Day Out develops diarrhea, weakness, abdominal distention, flatulence, abdominal cramps, and foul smelling greasy stools. Treatment?

Dx is Giardia Treatment is Tinidazole or Metronidazole

How to treat labial adhesions of the labia minora?

Estrogen cream daily for a week and then petrolatum for a month or two after separation.

Neural tube defects are associated with deficiency in which nutrient?

Folate deficiency

What enzyme is deficient in galactosemia?

Galactose-l-phosphate uridyl transferase, results in a block in the metabolic pathway of galactose and leads to the accumulation of galactose-l- phosphate in the tissues. Infants with this condition develop serious damage to liver, brain, and eyes after being fed milk containing lactose (a disaccharide compound of glucose and galactose). Clinical manifestations include lethargy, vomiting and diarrhea, hypotonia, hepatomegaly and jaundice, failure to thrive, and cataracts. Most untreated patients develop physical and mental retardation. Treatment consists of prompt elimination of lactose-containing milk from the diet in infancy and, as a more varied diet is introduced, exclusion of foods that contain casein, dry milk solids, whey, or curds.

Most common cause for hypoglycemia in SGA infants?

Glycogen and fat stores are diminished in premature infants and those who are small for gestational age. Energy stores are inadequate to meet the energy demands after the maternal supply of glucose is interrupted at birth, and hypoglycemia ensues. Deficiency of cortisol or growth hormone is a rare cause of neonatal hypoglycemia.

Name the two ways that acute osteomyeilitis can occur.

Hematogenous spread External trauma (including deep cellulitis).

What am I? A child with intractable chronic constipation without fecal soiling with a neonatal history of delayed passage of meconium, and the infant can continue to be constipated with bouts of abdominal distention and vomiting.

Hirschsprung Disease

474. A developmentally delayed 2-year-old has bilateral hypopigmented whorls on the upper extremities

Hypomelanosis of Ito is not thought to be inherited, but does display mosaicism in that about half of affected patients have two distinct cell lines of skin fibroblasts. Other examples of inherited mosaicism include higher functioning patients with trisomy 21, who may display some physical findings consistent with the syndrome but may not have as severe cognitive delay, as some cells have trisomy 21 and others do not.

Describe the presentation of tricyclic antidepressant ingestion.

In smaller children, the central nervous system (CNS) symptoms of *drowsiness, lethargy, coma, and seizures* are more commonly seen than the cardiac effects of tachycardia, initial hypertension followed by hypotension, *widening of the QRS* complex and ventricular dysrhythmias, which are often seen in adolescents. While therapy is mostly supportive, patients with a QRS complex wider than 100 ms, intractable hypotension, or ventricular dysrhythmias are candidates for alkalinization with an initial 1-2 mEq/kg of sodium bicarbonate.

First step in treating a diaphragmatic hernia?

In the neonate, respiratory failure in the first hours of life, a scaphoid abdomen, and presence of bowel sounds in the chest are common findings. Intensive respiratory support, including high-frequency oscillatory ventilation and extracorporeal membrane oxygenation (ECMO), has increased survival.

What are the levels of phosphate and calcium in vitamin D deficiency?

Intestinal absorption of calcium and phosphorus is diminished in vitamin D deficiency. Transient hypocalcemia stimulates the secretion of parathyroid hormone and the mobilization of calcium and phosphorus from bone; enhanced parathyroid hormone activity leads to phosphaturia and diminished excretion of calcium.

365. A 4-year-old child presents in the clinic with an illness notable for swelling in front of and in back of the ear on the affected side, as well as altered taste sensation. What organ systems can be involved with this? What is the illness?

Mumps c. Involvement of the central nervous system (CNS) may occur 10 days after the resolution of the swelling. Meningitis, pancreatitis, and renal involvement can occur as part of the disease. Many patients with mumps have some WBCs in their spinal fluid. The meningitis that occurs with mumps can occur at the same time as the parotitis, or following the parotitis by about 10 days.

Name 3 drugs contraindicated in Breast feeding?

Lipid soluble. Lithium, cyclosporin, antineoplastic agents, cocaine and heroin, amphetamines, ergotamines, and bromocriptine.

How to manage Vesiculoureteral Reflux (VUR)?

Low dose antibiotics if reflux for mild to moderate VUR. Surgery for severe VUR (dilated and tortuous ureter).

369. A 10-year-old boy from the Connecticut coast is seen because of discomfort in his right knee. He had a large, annular, erythematous lesion on his back that disappeared 4 weeks prior to the present visit. His mother recalls pulling a small tick off his back. What are some other presentations of this disease?

Lyme disease In addition to skin and joint involvement, CNS and cardiac abnormalities may be present.

Describe the clinical presentation of an infant with congenital syphilis.

Maculopapular peeling rash on the face, palms, and soles. Rhinitis resultant serous, and occasionally purulent, blood-tinged discharge (snuffles). Saddle nose, a result of destruction of bone from syphilitic rhinitis. Hepatosplenomegaly and lymphadenopathy are common. Among the later manifestations, or stigmata, of congenital syphilis is interstitial keratitis, which is an acute inflammation of the cornea that begins in early childhood (most commonly between 6 and 14 years of age). Interstitial keratitis represents the response of the tissue to earlier sensitization. Findings include marked photophobia, lacrimation, corneal haziness, and eventual scarring. Hutchinson teeth (peg or barrel-shaped upper central incisors), abnormal enamel, and mulberry molars (first lower molars with an abnormal number of cusps) are dental manifestations of syphilis.

What does the curly Q sign on barium x-ray indicate?

Malrotation with volvulus! "curly Q" twist of barium as it passes through the malrotated portion of bowel.

A 14-year-old girl with a history of precocious puberty who now develops a large goiter and unilateral cafe-au-lait macules, polyostotic fibrous dysplasia

McCune Albright

Treatment for trichamonas

Metronidazole Tinidazole

117. A recovering premature infant who weighs 950 g (2 lb, 1 oz) is fed breast milk to provide 120 cal/kg/d. Over the ensuing weeks, the baby is most apt to develop which of the following?

b. Hypocalcemia

An 18-year-old with a history of fractures and optic gliomas and caufe-au-lait spots.

Neurofibromatosis type 1

A 900-gram premature infant born at 28 weeks gestation has a continuous machinery-like murmur at the second left intercostal space that radiates well to the anterior lung fields but not to the back. Cause of the murmur?

PDA The ductus arteriosis usually closes spontaneously shortly after delivery in a term infant in response to movement from placental support to the relatively higher oxygen concentration in room air. In preterm infants, however, the smooth muscle in the wall of the ductus is not so responsive to increased oxygen. Although the PDA can be helpful in the very few patients with cyanotic congenital heart disease, it can lead to complications for most premature infants.

What is the most common finding in rheumatic fever?

Polyarthritis But any of the JONE's criteria can appear.

What is prune belly syndrome?

Prune belly syndrome, a malformation that occurs mostly in males, is characterized by a lax, wrinkled abdominal wall, a dilated urinary tract, and intra-abdominal testicular tissue. Additional urinary tract abnormalities include significant renal dysfunction or dysplasia. Oligohydramnios and commonly associated pulmonary complications, such as pulmonary hypoplasia and pneumothorax, are seen. Congenital hip dislocation, clubfeet, and intestinal malrotation with possible secondary volvulus can occur.

455. A large 5-day-old girl with brachydactyly, round facies, and short neck

Pseudohypoparathyroidism is a collective term for a variety of diseases. Affected children are short, round-faced, and mildly retarded. Metacarpals and metatarsals are shortened, and subcutaneous and basal ganglia calcifications as well as cataracts can be present. The current treatment consists of large doses of vitamin D and reduction of the phosphate load.

What is rumination?

Regurgitation, rechewing, then swallowing again. Rumination may also be seen in adolescents, and may be misdiagnosed as bulimia. While some studies have suggested an association, bulimic patients do not typically reswallow their food. Treat with behavioral therapy.

A 14-year-old Asian boy arrives via ambulance to your emergency room from the local international airport. He developed severe, intermittent abdominal pain that radiates into his scrotum about half-way through his 12-hour flight. He reports no fever, some dysuria, and he thinks he may have blood in his urine. What does the patient have?

The child in the case likely has urolithiasis. While renal stones in children are relatively uncommon in the United States and are usually related to metabolic abnormalities (where they are twice as common in boys), they are endemic in southeast Asia and related to diet. Symptoms of a stone in the renal pelvis, calyx, or ureter cause obstruction and the symptoms presented in the case. A stone in the distal ureter results in symptoms of dysuria, urgency, and frequency.

What level of dehydration does "doughy" refer to?

Severe, hypernatremic hypovolemia. (>10%)

A 6-month-old with blindness on the same side as a large facial lesion

Sturge-Webber (PHACE syndrome)

170. An infant presents with a large, flat vascular malformation over the left face and scalp. The mother notes that her other child was born with a capillary hemangioma on his arm and asks if this is the same thing. You explain that this vascular malformation is different, and that you will want to monitor him for another condition. What is the diagnosis?

Sturge-Weber syndrome A flat capillary vascular malformation in the distribution of the trigeminal nerve is the primary cutaneous lesion. The malformation also involves the meninges and results in atrophy to the underlying cerebral cortex. The damage is manifested clinically by seizures, mental deficiency, and hemiparesis or hemianopsia on the contralateral side. The cause is unknown.

Child has increased FOC and is squishy all over the scalp? What is it and how do you manage it?

Subglial hematoma, send to ICU!

339. A 6-year-old boy with eye twitching and echolalia.

Sydenham chorea The term "chorea" describes involuntary uncoordinated jerks of the arms and legs. Sydenham chorea is the most common acquired chorea of childhood, is seen after infections with group A β-hemolytic streptococci, and is associated with rheumatic heart disease and arthritis. In addition to the motor symptoms, patients may be hypotonic, emotionally labile, and have a "milkmaid" grip with sequential grip tightening and relaxing. Other findings include a darting tongue and "spooning" of an extended hand (flexion at the wrist and extension of the fingers).

A mother arrives to your office in a panic. She had just returned home from the delivery of her third child, who was born by cesarean section because he was large for gestation age. Upon changing his first diaper at home she noticed gross blood in his diaper. On examination, you find a left-sided abdominal mass that was not present in the normal newborn nursery. Which of the following is the most appropriate next step in the diagnosis and management of this child? a. Urine catheterization and initiation of intravenous antibiotics b. Measurement of urine catecholamines to include vanillylmandelic and homovanillic acid levels c. CT scan of the chest and abdomen d. Renal ultrasound with renal vein Doppler studies e. Intravenous pyelogram

The child in the question likely has renal vein thrombosis, a condition more commonly seen in infants who are dehydrated, had birth depression, have polycythemia or were born to diabetic mothers. The child in the case was large for gestational age, a hint that gestational diabetes might have been present and which also can result in polycythemia. Children with renal vein thrombosis can present with history of oliguria and hematuria, and on examination enlarged kidneys may be palpable. Confirmation of the diagnosis is by renal ultrasound with renal vein Doppler studies.

What happens to the heart in glycogen storage diseases?

With glycogen storage disease of the heart muscle, thickening would be expected and with pericarditis, a pericardial effusion would be present.

How to treat hydrocele in a new born?

The description is that of a hydrocele, an accumulation of fluid in the tunica vaginalis. Small hydroceles usually resolve spontaneously in the first year of life.

What are the first and second-line treatments for supraventricular tachycardia in infants?

The first-line treatment is to stimulate the vagus nerve using techniques such as carotid massage, immersion of the face in cold water, or voluntary straining. Rapid infusion of IV adenosine can affect resolution if the maneuvers are not successful.

334. A previously healthy 16-year-old girl presents to the emergency center with the complaint of "falling out." She was with her friends at a local fast food restaurant when she felt faint and, according to her friends, lost consciousness for about a minute. There was no seizure activity noted, but the friends did notice her arms twitching irregularly. She is now acting normally. She denies chest pain or palpitations, and her electrocardiogram (ECG) is normal. Diagnosis?

The majority of syncopal episodes are vasovagal. The sequence of events described in the question probably resulted from vasovagal stimulation, often precipitated by pain, fear, excitement, or standing for long periods. This condition is common in adolescent girls. Counseling on proper diet and fluid intake is appropriate; patients should also increase their salt intake. Occasionally, patients with recurrent vasovagal syncope may be managed using β-blockers or fludrocortisone.

Describe three common findings in infective endocarditis

The much discussed Osler nodes (small tender nodules in the tips of fingers and toes), Janeway lesions (nontender hemorrhagic lesions on the hands and feet), and splinter hemorrhages (dark lines under nails) are uncommon findings. Gram-positive organisms, particularly alpha-hemolytic streptococci (Streptococcus viridans), Staphylococcus aureus, and coagulase-negative staphylococci, are the most common offenders. S aureus is the most common cause of acute bacterial endocarditis.

How to treat child with suspected TB?

Treat the 3-year-old patient with isoniazid (INH) and rifampin

An infant with infantile spasms, a hypsarrhythmic EEG pattern, and ash-leaf depigmentation on her back

Tuberous sclerosis

Definition of monochorionic diamnionic?

Twins are in two different amniotic sacs but receive nutrition from one placenta. Monochorionic is the only way that twin to twin transfusion can occur.

How to manage child with sickle-cell with evidence of a stroke.

Urgent transfusion of blood to reduce the number of circulating sickled cells, and hospitalization (likely in the intensive care unit) to observe for further neurologic deterioration are indicated. As recovery begins, physical therapy is instituted and the patient is enrolled in a chronic blood transfusion program to reduce the risk of recurrence.

169. An infant has fusion of the eyebrows, heterochromic irises, a broad nasal root with lateral displacement of the medial canthi, and a white forelock What is the diagnosis?

Waardenburg syndrome is inherited as an autosomal dominant trait with variable penetrance. It includes: lateral displacement of the medial canthi (the outer or inner corner of the eye, where the upper and lower lids meet.), broad nasal bridge, medial hyperplasia of the eyebrows, partial albinism commonly expressed by a white forelock or heterochromia (or both), and deafness in 20% of cases.

421. Friends are considering adopting a "special needs" child from another country. The family has few details, but the information they have received so far suggests the 4-year-old child has had surgery for an endocardial cushion defect, is short for his age, and had a history of what sounds like surgically repaired duodenal atresia at birth. You are suspicious this child may have which of the following syndromes?

Waardenburg syndrome is the most common of several syndromes that are characterized by both deafness and pigmentary changes. Features of this syndrome, which is inherited as an autosomal dominant disorder, include a distinctive white forelock, heterochromia irides, unilateral or bilateral congenital deafness, and lateral displacement of the inner canthi.

What is Harlequin syndrome?

a transient change in the skin color of the otherwise asymptomatic newborn (usually preterm) in which the dependent side of the entire body turns red while the upper side remains pale.

2. An infant can regard his parent's face, follow to the midline, lift his head from the examining table, smile spontaneously, and respond to a bell. He does not yet regard his own hand, follow past the midline, nor lift his head to a 45° angle off the examining table. Which of the following is the most likely age of the infant?

a. 1 month

81. Vitamin C

a. A 4-year-old with diarrhea, abdominal pain, and kidney stones which prove to be caused by calcium oxalate.

30. You find a discrete, whitish polyp that extends through the tympanic membrane in a child with a history of recurrent otitis media. This most likely represents which of the following?

a. A cholesteatoma

265. A 7-day-old premature infant born at 26 weeks of gestation now has a grossly bloody stool, abdominal distention, and increasing oxygen requirements.

a. Abdominal series

99. Liver disease What supplement is indicated for baby?

a. All fat-soluble vitamins

483. You are the sideline physician for a local high school football team. During a district playoff game, the starting quarterback is sacked for a loss on third down. As the punter heads out onto the field, the quarterback is slow to come to the sidelines. He seems confused and dazed. Aside from his confusion, his examination is normal. After 10 minutes, he is lucid and wants to get back into the game. Based on published guidelines, which of the following is your correct course of action?

a. Allow the player back in the game The guidelines divide concussions into three categories. Grade 1 (mild) concussions have no loss of consciousness and a return to baseline in less than 15 minutes. Grade 2 (moderate) concussions also have no loss of consciousness, but include confusion lasting longer than 15 minutes. Grade 3 (severe) concussions include any loss of consciousness. The player in the vignette had a grade 1 concussion with no other sign of injury, and as such may be allowed back into the game. If, later in the game, he were to have a second grade 1 concussion, he should be removed from the game and held out of play for a week. A player with a grade 2 concussion may not return to the game and must remain out for at least a week; he should be evaluated in a hospital if symptoms last for more than an hour. A player with a grade 3 concussion should be evaluated immediately at the hospital.

447. Three boys in a family with hypodense calcification of their bones and a history of fractures; their condition is unresponsive to dietary changes or vitamin supplementation. What are the PO4 and Ca levels in this patient?

a. Low PO4, normal Ca The boys in the first question have rickets unresponsive to vitamin supplementation; vitamin D-resistant rickets is caused by a genetic abnormality in the renal tubular reabsorption of phosphate with resultant hyperphosphaturia and hypophosphatemia. Calcium concentration is usually normal.

300. A 2-year-old patient arrives late to your office with his father and a sign-language translator. They are very apologetic, but the father communicates that he had car trouble at his dialysis center and thus was late picking up the child from day care. The father is concerned about his child's having intermittent red, bloody-looking urine. A gross inspection of the child's urine in your office looks normal, but the dipstick demonstrates 3+ blood. Which of the following is the most likely cause of this child's hematuria?

a. Alport syndrome The most common type of hereditary nephritis is Alport syndrome. Clinically, patients present with asymptomatic microscopic hematuria, but gross hematuria is also possible, especially after an upper respiratory infection. Hearing loss, eventually leading to deafness, is associated with Alport syndrome in up to 75% of cases.

11. A previously healthy 5-year-old boy has a 1-day history of low-grade fever, colicky abdominal pain, and a rash. He is well-appearing and alert. His vital signs, other than a temperature of 38°C (100.5°F) are completely normal. A diffuse, erythematous, maculopapular, and petechial rash is present on his buttocks and lower extremities, as shown in the photograph. He has no localized abdominal tenderness or rebound; bowel sounds are active. Laboratory data demonstrate Urinalysis: 30 red blood cells (RBCs) per high-powered field, 2+ protein Stool: Guaiac positive Platelet count: 135,000/μL These findings are most consistent with which of the following?

a. Anaphylactoid purpura

75. After helping his father in the yard, a 14-year-old boy complains of weakness and feels like his muscles are twitching. He begins to drool, and then collapses in a generalized tonic-clonic seizure. Upon the arrival of EMS, his heart rate is found to be 40 beats per min and his pupils are pinpoint

a. Atropine and pralidoxime (2-PAM)

199. The mother has a history of systemic lupus erythematosus and gestational diabetes. The nurses are concerned because the baby has developed petechiae and bruising after his bath. Vital signs have been stable, with a heart rate in the 60 beats per minute range and a respiratory rate in the 40 breaths per minute range. You note a large liver, scattered petechiae, and an erythematous rash on the cheeks and on the bridge of the nose. What is her heart condition?

a. Congenital heart block Neonatal lupus is a rare manifestation of transferred maternal IgG autoantibodies. Infants can have thrombocytopenia, neutropenia, rash, liver dysfunction, and a congenital heart block. Most manifestations are self-resolved; however, the congenital heart block is permanent and frequently requires pacing.

144. The mother of a 2-week-old infant reports that since birth, her infant sleeps most of the day; she has to awaken her every 4 hours to feed, and she will take only an ounce of formula at a time. On your examination you find an infant with normal weight and length, but with an enlarged head. The heart rate is 75 beats per minute and the temperature is 35°C (95°F). The child is still jaundiced. You note large anterior and posterior fontanelles, a distended abdomen, and an umbilical hernia. This clinical presentation is likely a result of which of the following?

a. Congenital hypothyroidism Umbilical hernia, cool temperature all point toward hypothyroidism. Sepsis can cause hypothermia and poor feeding, but the 2-week course makes this choice unlikely.

158. A newborn has hydrocephalus, chorioretinitis, intracranial calcifications, and anemia.

a. Congenital toxoplasmosis CMV is the most common sort of congenital infection, with infection estimates ranging from 0.4% to 2.4% of all live births. Multiorgan manifestation of disease including intrauterine growth restriction (IUGR), hepatosplenomegaly, jaundice, petechiae or purpura, microcephaly, chorioretinitis, and intracranial calcifications. More than half of infants with this congenital infection develop sensorineural hearing loss.

237. An 8-year-old boy presents to your office for a second opinion. He has a 2-year history of intermittent vomiting, dysphagia, and epigastric pain. His father reports he occasionally gets food "stuck" in his throat. He has been on a proton pump inhibitor for 18 months without symptom relief. His past history is significant only for eczema and a peanut allergy. Endoscopy was performed 6 months ago; no erosive lesions were noted and a biopsy was not performed. You arrange for a repeat endoscopy with biopsy. Microscopy on the biopsy sample reveals many eosinophils. Treatment of this condition should include which of the following?

a. Corticosteroids Dx: Eosinophilic esophagitis (an allergic response). Males are affected more than females. The history usually includes atopy or food allergy. Symptoms are similar to those seen in gas-troesophageal reflex disease, but are not relieved with acid blockade. Some have elevated IgE levels or peripheral eosinophilia (hinted at by his history of eczema and peanut allergy). Endoscopy reveals mucosal furrowing; strictures can develop as well. Biopsy reveals many eosinophils (normal mucosa does not have eosinophils). Treatment includes avoidance of specific food allergens. Inhaled or systemic steroids have been helpful.

Questions 492 to 495 492. A 16-year-old female has a positive rapid plasma reagin (RPR) of 1:64 and an ulcer on her labia.

a. Dark-field microscopic examination

77. A 2-year-old boy found a bottle of his mother's prenatal vitamins and consumed the majority of them. He now has hematemesis and abdominal pain. He is febrile, and laboratory tests reveal a leukocytosis and hyperglycemia

a. Deferoxamine mesylate Deferoxamine given intravenously or intramuscularly forms a complex with the iron and is excreted in the urine, to which it imparts the color of vin rosé (red wine).

34. A 20-month-old child is brought to the ED because of fever and irritability and refusal to move his right lower extremity. Physical examination reveals a swollen and tender right knee that resists passive motion. Which of the following is the most likely to yield the diagnosis in this patient?

a. Examination of joint fluid

16. A 10-year-old child arrives with the complaint of new-onset bed-wetting. He has had no fever, his urine culture is negative, and he has had no new stresses in his life. He is well above the 95th percentile for weight as is much of his family. Which of the following is most helpful in making a diagnosis?

a. Fasting plasma glucose of 135 mg/dL

62. Megaloblastic anemia, glossitis, pharyngeal ulcers, impaired immunity

a. Folate deficiency

92. A 4-month-old presents with a dry, scaly rash on his cheeks, arms, and upper chest. His 10-year-old sister had a similar rash when she was young, but the rash is now confined to her antecubital and popliteal fossa; her rash worsens in winter months

a. Mild cleansing cream, topical moisturizers, and topical steroids Atopic dermatitis

208. A previously healthy, active, 18-month-old African American child presents with unilateral nasal obstruction and foul-smelling discharge. The child's examination is otherwise unremarkable. Which of the following is the most likely diagnosis?

a. Foreign body Small children frequently introduce any number of small objects into their noses, ranging from food to small toys. Initially, only local irritation occurs. Later, as prolonged obstruction is seen, symptoms increase to include worsening of pain, and a purulent, malodorous, bloody discharge can be seen. Unilateral nasal discharge in the presence of obstruction suggests the need to examine the patient for a nasal foreign body.

55. A teen, just back from a skiing trip, has blistering and peeling of several areas on her face; she reports the lesions started as firm, cold, white areas that felt stinging at the time and are now more sensitive than the surrounding skin.

a. Frostnip

94. A 14-year-old child has headache, hypertension, edema, and a change in urine output and color

a. Glomerulonephritis

478. A 15-year-old girl is brought to the pediatric emergency room by the lunchroom teacher, who observed her sitting alone and crying. On questioning, the teacher learned that the girl had taken five unidentified tablets after having had an argument with her mother about a boyfriend of whom the mother disapproved. Toxicology studies are negative, and physical examination is normal. Which of the following is the most appropriate course of action?

a. Hospitalize the teenager on the adolescent ward. The adolescent who has attempted suicide should be hospitalized so that a complete medical, psychological, and social evaluation can be performed and an appropriate treatment plan developed. Hospitalization also emphasizes the seriousness of the adolescent's action to her and to her family and the importance of cooperation in carrying out the recommendations for ongoing future therapy. The treatment plan may include continued counseling or supportive therapy with a pediatrician, outpatient psychotherapy with a psychiatrist or other mental health worker, or family therapy.

174. Previous premature infant born at 27 weeks' gestation and now 6 months of age presents with macrocephaly and hydrocephalus on ultrasonogram

a. Intraventricular hemorrhage

211. A fully immunized 2-year-old presents to the emergency room with several days of low-grade fever, barking cough, and noisy breathing. Over the past few hours he has developed a fever of 40°C (104°F) and looks toxic. He has inspiratory and expiratory stridor. The family has not noticed drooling, and he seems to be drinking without pain. Direct laryngoscopy reveals a normal epiglottis. The management of this disease process includes which of the following?

a. Intubation and intravenous antibiotics Bacterial tracheitis is an uncommon but severe and life-threatening sequela of viral laryngotracheobronchitis. The typical story is that presented in the case, with several days of viral upper respiratory symptoms, followed by an acute elevation of temperature and an increase in respiratory distress. Inspiratory stridor is typical in croup; the biphasic stridor and high fever in this patient should be a clue to consider alternative diagnoses. Management for tracheitis includes establishing an airway with endotracheal intubation and IV antibiotics. Special attention is focused on preservation of the airway, as even intubated children with tracheitis can have secretions thick and copious enough to occlude the airway.

129. At 43 weeks' gestation, a long, thin infant is delivered. The infant is apneic, limp, pale, and covered with "pea soup" amniotic fluid. Which of the following is the best first step in the resuscitation of this infant at delivery?

a. Intubation and suction of the trachea; provision of oxygen Infants who are postdates (more than 42 weeks' gestation) and show evidence of chronic placental insufficiency (low birth weight for gestational age and wasted appearance) have a higher than average chance of being asphyxiated, and passage of meconium into the amniotic fluid places these infants at risk for meconium aspiration.

480. A 17-year-old sexually active girl comes to your office complaining of acne that is unresponsive to the usual treatment regimen. Physical examination reveals severe nodulocystic acne of her face, upper chest, and back. You consider prescribing isotretinoin (Accutane), but you are concerned about side effects. Reviewing the literature, you find which of the following to be true about isotretinoin?

a. Its efficacy can be profound and permanent. Isotretinoin (13-cis-retinoic acid; Accutane) has proved to be very effective in the treatment of refractory nodulocystic acne. The effects of treatment appear to be long-lasting. Precautions regarding its use, however, are essential. Because of its teratogenic effects (isotretinoin syndrome), the drug is contraindicated during pregnancy and within 1 month of becoming pregnant. Dry skin, eyes, and mucous membranes are the most frequent complications of therapy. Other associated problems include musculoskeletal pain and hyperostosis, inflammatory bowel disease, pseudotumor cerebri, and corneal opacities. Patients on isotretinoin therapy can often develop abnormal liver function tests, elevated triglyceride and cholesterol levels, and lowered levels of high-density lipoproteins. Some have suggested that an increased risk of suicide is related to its use, although this link is less well established.

4. A 4-year-old girl is noticed by her grandmother to have a limp and a somewhat swollen left knee. The parents report that the patient occasionally complains of pain in that knee. An ophthalmologic examination reveals findings as depicted in the photograph. Which of the following conditions is most likely to be associated with these findings?

a. Juvenile rheumatoid arthritis

397. After being delivered following a benign gestation, a newborn infant is noted to have a platelet count of 35,000/μL, decreased fibrinogen, and elevated fibrin spilt products. On examination you note a large cutaneous hemangioma on the abdomen that is purple and firm. Which of the following anomalies might also be expected in this infant?

a. Kaposiform hemangioendothelioma The child in the question likely has Kasabach-Merritt phenomenon, which is seen with large vascular anomalies (ie, Kaposiform hemangioendothelioma and tufted angioma). Platelet and RBC sequestration within the vascular tumor causes peripheral thrombocytopenia, coagulopathy, and microangiopathic hemolytic anemia. Treatment options include corticosteroids, α-interferon, and vincristine. Surgery frequently results in excessive bleeding.

68. A 6-year-old boy has developed a limp and has limited mobility of the hip, but denies pain and fever.

a. Legg-Calvé-Perthes disease

Questions 471 to 474 471. A 3-year-old boy with myoclonus, ataxia, weakness, and seizures who has cytochrome oxidase-negative ragged red fibers noted on muscle biopsy

a. Mitochondrial inheritance

203. A previously well 1-year-old infant has had a runny nose and has been sneezing and coughing for 2 days. Two other members of the family had similar symptoms. Four hours ago, his cough became much worse. On physical examination, he is in moderate respiratory distress with nasal flaring, hyperexpansion of the chest, and easily audible wheezing without rales. His chest radiographs are shown. Which of the following is the appropriate next course of action?

a. Monitoring oxygenation and fluid status alone Of the choices given and with the findings on the radiograph (patchy infiltrates with flat diaphragms), monitoring oxygenation and hydration status is the most appropriate course of action as bronchiolitis is the most likely diagnosis.

A 14-year-old girl with low-grade fever, cough of 3 weeks' duration, and interstitial infiltrate.

a. Mycoplasmal pneumonia Mycoplasma pneumoniae is a common cause of pneumonia in the *school-age child* or *young adult*. Usual presentation includes the gradual onset of headache, malaise, fever, and lower respiratory symptoms. Typically, the cough (often nonproductive) worsens for the first 2 weeks of the illness, and then slowly resolves over the ensuing 3 to 4 weeks.

184. An ill-appearing 2-week-old baby girl is brought to the emergency room. She is pale and dyspneic with a respiratory rate of 80 breaths per minute. Heart rate is 195 beats per minute, heart sounds are distant, a gallop is heard, and she has cardiomegaly on x-ray. An echocardiogram demonstrates poor ventricular function, dilated ventricles, and dilation of the left atrium. An ECG shows ventricular depolarization complexes that have low voltage. Which of the following is the most likely diagnosis based on this clinical picture?

a. Myocarditis The findings of pallor, dyspnea, tachypnea, tachycardia, and cardiomegaly are common in congestive heart failure regardless of the cause. The constellation of findings in the question suggests myocarditis as the etiology of this patient's condition. The most common causes of myocarditis include adenovirus and coxsackievirus B, although many other viruses can cause this condition.

462. An 8-year-old boy with Addison disease (in crisis)

a. Na+ 118, K+ 7.5 Addison disease is associated with a combined deficiency of glucocorticoids and mineralocorticoids. Resorption of sodium and excretion of potassium and hydrogen ions are impaired at the level of the distal renal tubules. Sodium loss results in loss of water and depletion of blood volume. Persons with compensated Addison disease can have relatively normal physical and laboratory findings; Addisonian crisis, however, characteristically produces hyponatremia, hyperkalemia, and shock. The pathophysiology of the serum electrolyte abnormalities in this disorder is the same as in the salt-losing variety of adrenogenital syndrome.

305. Elevated levels of cholesterol and triglycerides found in a 6-year-old boy whose mother reports that he has been awakening with puffy eyes each morning. On your physical examination you determine that he has had unexpected weight gain and has scrotal edema

a. Nephrotic syndrome

Questions 49 to 53 49. A 3-year-old boy awakens every night around 2:00 AM screaming incoherently. His parents note that he is agitated, seems awake but unresponsive, and goes back to sleep within a few minutes. He has no memory of the episodes in the morning

a. Night terrors

425. A 6-year-old boy is brought to your practice by his paternal grandmother for his first visit. She has recently received custody of him after his mother entered the penal system in another state; she does not have much information about him. You note that the child is short for his age, has downslanting palpebral fissures, ptosis, low-set and malformed ears, a broad and webbed neck, shield chest, and cryptorchidism. You hear a systolic ejection murmur in the pulmonic region. His grandmother reports that he does well in regular classes, but has been diagnosed with learning disabilities and receives speech therapy for language delay. His constellation of symptoms is suggestive of which of the following?

a. Noonan syndrome The constellation of signs described suggest Noonan syndrome. Other features of this syndrome include cubitus valgus, pulmonary stenosis, edema of the dorsum of the hands and feet, hearing loss, pectus excavatum, bleeding diathesis, and mental retardation in about one-fourth of cases. These patients have many features in common with Turner syndrome, and the condition is often referred to as the "male Turner syndrome," although it occurs in both genders.

21. A 16-year-old arrives to your office soon after beginning basketball season. He states that he has had progressive pain in his knees. A physical examination reveals, in addition to tenderness, a swollen and prominent tibial tubercle. Radiographs of the area are unremarkable. Which of the following is the most likely diagnosis?

a. Osgood-Schlatter disease

236. A 10-year-old boy has been having "bellyaches" for about 2 years. They occur at night as well as during the day. Occasionally, he vomits after the onset of pain. Occult blood has been found in his stool. His father also gets frequent, nonspecific stomachaches. Which of the following is the most likely diagnosis?

a. Peptic ulcer the presence of nocturnal abdominal pain and GI bleeding in a patient with a positive family history supports a diagnosis of peptic ulcer disease (PUD).

359. A 14-month-old infant suddenly develops a fever of 40.2°C (104.4°F). Physical examination shows an alert, active infant who drinks milk eagerly. No physical abnormalities are noted. The WBC count is 22,000/μL with 78% polymorphonuclear leukocytes, 18% of which are band forms. Which of the following is the most likely diagnosis?

a. Pneumococcal bacteremia In an infant who appears otherwise normal, the sudden onset of high fever, together with a marked elevation and shift to the left of the WBC count, suggests pneumococcal bacteremia. The incidence of pneumococcal disease producing this picture may be decreasing with the widespread use of a pneumococcal vaccine.

319. An irritable 6-year-old child has a somewhat unsteady but nonspecific gait. Physical examination reveals a very mild left facial weakness, brisk stretch reflexes in all four extremities, bilateral extensor plantar responses (Babinski reflex), and mild hypertonicity of the left upper and lower extremities; there is no muscular weakness. Which of the following is the most likely diagnosis?

a. Pontine glioma A child who has a subacute disorder of the CNS that produces cranial nerve abnormalities (especially of cranial nerve VII and the lower bulbar nerves), and unsteady gait secondary to spasticity is most likely to have a pontine glioma.

A large 6-day-old infant with severe hypotonia and poor feeding since birth What is the diagnosis?

a. Prader-Willi syndrome The Prader-Willi syndrome is a disorder consisting of hypotonia, hypogonadism, hyperphagia after the newborn period, hypomentia, and obesity. A deletion of a portion of chromosome 15 has been found in approximately 70% of patients. Children affected by this syndrome exhibit little movement in utero and are hypotonic during the neonatal period. Feeding difficulties and failure to thrive can be the presenting complaints in the first year; later, obesity becomes the most common presenting complaint. The enormous food intake of affected children is thought to be caused by a defect in the satiety center in the hypothalamus. Stringent caloric restriction is the only known treatment.

287. A 5-year-old girl without past history of UTI is in the hospital on antibiotics for Escherichia coli pyelonephritis. She is still febrile after 4 days of appropriate antibiotics. A renal ultrasound revealed no abscess, but a focal enlargement of one of the lobes of the right kidney. CT of the abdomen reveals a wedgeshaped area in the right kidney distinct from the normal tissue with minimal contrast enhancement. Appropriate management of this patient includes which of the following interventions?

a. Prolonged antibiotic therapy The patient in the question has acute lobar nephronia, which is in the middle of the spectrum between pyelonephritis and renal abscess. Patients have prolonged fever curves despite appropriate antibiotics. A CT scan is most useful in diagnosing nephronia, but a renal ultrasound can identify the process as well. Treatment is prolonged IV and then PO antibiotics. Shorter courses like the typical treatment for pyelonephritis can lead to recurrence and renal abscess.

139. You are called to a delivery of a term infant, about to be born via cesarean section to a mother with multiple medical problems, including a 1-month history of a seizure disorder, for which she takes phenytoin; rheumatic heart disease, for which she must take penicillin daily for life; hypertension, for which she takes propranolol; acid reflux, for which she takes aluminum hydroxide; and a deep venous thrombosis in her left calf diagnosed 2 days ago, for which she was started on a heparin infusion. The obstetrician is concerned about the possible effects of the mother's multiple medications on the newborn infant. Which of the following medications is most likely to cause harm in this newborn infant at delivery?

a. Propranolol Phenytoin may cause birth defects when given during the *first trimester*. Propranolol, which may cause growth retardation when given throughout pregnancy, diminishes the ability of an asphyxiated infant to increase heart rate and cardiac output. It has also been associated with hypoglycemia and apnea.

364. A 14-year-old girl awakens with a mild sore throat, low-grade fever, and a diffuse maculopapular rash. During the next 24 hours, she develops tender swelling of her wrists and redness of her eyes. In addition, her physician notes mild tenderness and marked swelling of her posterior cervical and occipital lymph nodes. Four days after the onset of her illness, the rash has vanished. Which of the following is the most likely diagnosis?

a. Rubella

178. The parents of a 2-month-old baby boy are concerned about his risk of coronary artery disease because of the recent death of his 40-year-old maternal uncle from a myocardial infarction. Which of the following is the most appropriate management in this situation?

a. Screen the parents for total cholesterol.

46. The obstetrical resident on call asks you to evaluate an area of a newborn's scalp that seems to have no hair and is scaly and yellowish

a. Sebaceous nevus

244. An 8-year-old is accidentally hit in the abdomen by a baseball bat. After several minutes of discomfort, he seems to be fine. Over the ensuing 24 hours, however, he develops a fever, abdominal pain radiating to the back, and persistent vomiting. On examination, the child appears quite uncomfortable. The abdomen is tender, with decreased bowel sounds throughout, but especially painful in the midepigastric region with guarding. Which of the following tests is most likely to confirm the diagnosis?

a. Serum amylase levels While no diagnostic test is completely accurate, an elevated total serum amylase with the correct clinical history and signs and symptoms of *pancreatitis* is the best diagnostic tool.

256. You are admitting to the hospital a 3-month-old infant who has been having poor feeding, emesis, and diarrhea for 3 days. In the emergency center, her electrolytes were found to be: sodium 157 mEq/L, potassium 2.6 mEq/L, chloride 120 mEq/L, bicarbonate 14 mEq/L, creatinine 1.8 mEq/L, blood urea nitrogen (BUN) 68 mEq/L, and glucose 195 mEq/L. She was given a fluid bolus in the emergency center and has subsequently produced urine. Which of the following is the most appropriate next step in her management?

a. Slow rehydration over 48 hours Slow correction of this hypernatremia (0.5 mEq/L per hour over several days) prevents significant fluid shifts in the brain that may result in increased intracranial pressure and herniation. Hyperglycemia may be seen in hypernatremic dehydration because of decreased insulin secretion and cell sensitivity to insulin.

40. As part of your anticipatory guidance to new parents of a healthy newborn, you suggest putting the child in which of the following positions for sleep?

a. Supine position

337. A 12-year-old boy has chronic headache that worsens during the school day. These headaches are not associated with nausea or emesis, and he does not have any symptoms prior to the headache.

a. Tension headache Tension headaches are common in the older child and adolescent. They will worsen during the day, and may worsen with stressful situations like tests. They are typically described as squeezing, but are not usually pulsatile. Nausea and vomiting are not typical.

114. A 2-year-old boy is being followed for congenital cytomegalovirus (CMV) infection. He is deaf and developmentally delayed. The child's mother informs you that she has just become pregnant and is concerned that the new baby will be infected and may develop serious consequences. Will the second child be affected?

a. The mother has antibodies to CMV that are passed to the fetus In the United States, 70% to 90% of adult women have serologic evidence of a past infection with CMV. Symptomatic congenital disease usually occurs when a mother has a primary CMV infection in the first trimester of pregnancy. Many of these babies die, and those who survive are severely affected. In the event of reactivation of CMV infection during pregnancy, maternal IgG, passed transplacentally, protects the infant from serious infection.

122. A 3-day-old infant, born at 32 weeks' gestation and weighing 1700 g (3 lb, 12 oz), has three episodes of apnea, each lasting 20 to 25 seconds and occurring after a feeding. During these episodes, the heart rate drops from 140 to 100 beats per minute, and the child remains motionless; between episodes, however, the child displays normal activity. Blood sugar is 50 mg/dL and serum calcium is normal. What is the reason for the apneic episodes?

a. They are due to an immature respiratory center Apneic episodes are characterized by an absence of respirations for more than 20 seconds and may be accompanied by bradycardia and cyanosis. A large number of conditions can cause central apnea. In an otherwise well premature infant, apnea is thought to be secondary to an incompletely developed respiratory center.

Questions 338 to 340 338. Eye blinking or throat-clearing noises in an otherwise healthy 8-yearold boy.

a. Transient tic disorder of childhood Transient tic disorder is the most common and is seen more often in boys; a family history is often noted. In this condition, the patient has eye blinking, facial movements, or throat clearing lasting for weeks to about a year. No medications are needed.

28. A mother arrives to the clinic with her three children (ages 2 months, 18 months, and 36 months). The 18-month-old has an intensely pruritic scalp, especially in the occipital region, with 0.5-mm lesions noted at the base of hair shafts, as shown in the picture. Which of the following therapies should be avoided in this situation?

a. Treatment of all household contacts with 1% lindane (Kwell)

Questions 63 to 66 63. A newborn infant is noted to have microcephaly with sloping forehead, cutis aplasia on the scalp, microphthalmia, and cleft lip and palate. His echocardiogram demonstrates a complex heart lesion including atrial septal defect (ASD), ventricular septal defect (VSD), and dextrocardia.

a. Trisomy 13

465. A 6-year-old girl with cognitive delay, a blowing systolic heart murmur, short stature, round face, bilateral transpalmar crease, upslanting palpebral fissures, small ears, and epicanthal folds.

a. Trisomy 21 (Down syndrome)

374. A 2-month-old infant comes to the emergency center with fever for 2 days, emesis, a petechial rash, and increasing lethargy. In the ambulance he had a 3-minute generalized tonic/clonic seizure that was aborted with lorazepam. He does not respond when blood is drawn or when an IV is placed, but he continues to ooze blood from the skin puncture sites. On examination, his anterior fontanelle is open and bulging. His CBC shows a WBC of 30,000 cells/μL with 20% band forms. Which of the infant's problems listed below is a contraindication to lumbar puncture?

a. Uncorrected bleeding diathesis The importance and urgency of the lumbar puncture in cases of suspected meningitis outweigh the usual niceties in the performance of procedures. Infants and children require adequate restraints, preferably local anesthesia, and sometimes sedation. Contraindications are few and include increased intracranial pressure in the patient without an open fontanelle that can result in herniation; severe cardiorespiratory distress; skin infection at the puncture site; and severe thrombocytopenia or other coagulation disorder, suggested by the oozing IV and venipuncture sites.

273. A 4-year-old whose diet consists mostly of cheese puffs and cola begins to have problems walking at night, complaining that he cannot see well. In addition, his skin has become dry and scaly, and he has complained of headache for a month.

a. Vitamin A

181. A newborn is diagnosed with congenital heart disease. You counsel the family that the incidence of heart disease in future children is which of the following?

b. 2% to 6% The incidence of congenital heart disease in the population is about 1%. The risk of congenital heart disease in a family with one child born with heart disease varies depending on the type of lesion the first child had but, overall, the rate averages from about 2% to 6% if the heart disease is not associated with a diagnosable syndrome.

277. A newborn with arching of the back temporally related to feeds but no emesis

b. 24-hour pH probe The third child may have gastroesophageal reflux, causing Sandifer syndrome, a condition in which infants will arch and become tonic to protect their airway from the refluxing gastric contents. While an upper GI series can sometimes diagnose gastroesophageal reflux, esophageal pH probe is currently the preferred diagnostic test.

83. Pyridoxine

b. A 6-year-old who has developed ataxia and sensory neuropathy.

210. You admitted to the hospital the previous evening a 1-year-old boy who presented with cough, fever, and mild hypoxia. At the time of his admission, he had evidence of a right upper lobe consolidation on his chest radiograph. A blood culture has become positive in less than 24 hours for Staphylococcus aureus. Approximately 20 hours into his hospitalization, the nurse calls you because the child has acutely worsened over the previous few minutes, with markedly increased work in breathing, increasing oxygen requirement, and hypotension. As you move swiftly to the child's hospital room, you tell the nurse to order which of the following?

b. A large-bore needle and chest tube kit for aspiration of a probable tension pneumothorax Tension pneumothorax, a recognized complication of staphylococcal (and other) pneumonia, is caused by toxin production by the bacteria leading to rupture of the alveoli into the pleural space. Tension pneumothorax can be quickly lethal if not recognized and treated; this makes a high index of suspicion and prompt diagnosis mandatory. The other complications can also occur but do not require so prompt a response. Immediate action to relieve the pressure of a tension pneumothorax is essential, usually accomplished by inserting a needle or catheter into the second or third intercostal spaces in the midclavicular line while the patient is supine.

Name the heart condition 194. An infant in the neonatal intensive care unit (NICU) has shortened lower distal arms but with thumbs. Her platelet count is profoundly low.

b. ASD Thrombocytopenia-absent radius (TAR) syndrome is diagnosed in the newborn who demonstrates profound thrombocytopenia, bilateral absence of radius, and abnormally shaped thumbs. Cardiac lesions include TOF and ASD. About 40% of patients die in the newborn period as a result of low-platelet-induced bleeding.

308. The 7-year-old boy now in your office was last seen 2 weeks ago with a mild viral upper respiratory tract infection. Today, however, he presents with fever, ataxia, weakness, headache, and emesis. In the office he has a 3 minute left-sided tonic-clonic seizure. You send him to the hospital and order a magnetic resonance imaging (MRI) of the brain, the results of which show disseminated multifocal white matter lesions that enhance with contrast. This boy's likely diagnosis is which of the following?

b. Acute disseminated encephalomyelitis ADEM is an autoimmune demy-elinating disease seen in children less than 10 years of age. It may follow many different types of infections, including upper respiratory tract infections, varicella, mycoplasma, herpes simplex virus, rubella, rubeola, and mumps; it may also follow immunizations.

8. A mother calls you on the telephone and says that your 4-year-old son bit the hand of her 2-year-old son 48 hours previously. The area around the injury has become red, indurated, and swollen, and he has a temperature of 39.4°C (103°F). Which of the following is the most appropriate response?

b. Admit the child to the hospital immediately for surgical debridement and antibiotic treatment.

176. A 10-year-old boy, the star pitcher for the Salt Lake City Little League baseball team, had a sore throat about 2 weeks ago but did not tell anyone because he was afraid he would miss the play-offs. Since several children have been diagnosed with rheumatic fever in the area, his mother is worried that he may be at risk as well. You tell her that several criteria must be met to make the diagnosis but the most common finding is which of the following?

b. Arthralgia

321. A 6-year-old child is hospitalized for observation because of a short period of unconsciousness after a fall from a playground swing. He has developed unilateral pupillary dilatation, focal seizures, recurrence of depressed consciousness, and hemiplegia. Which of the following is the most appropriate management at this time? What is the most likely diagnosis?

b. CT scan Compression of cranial nerve III and distortion of the brainstem, resulting in unilateral pupillary dilatation, hemiplegia, focal seizures, and depressed consciousness, suggest a progressively enlarging mass, most likely an epidural hematoma.

252. A 6-week-old infant is admitted to the hospital with jaundice. Her outpatient blood work demonstrated a total bilirubin of 12 mg/dL with a direct portion of 3.5 mg/dL. Which of the following disorders is most likely to be responsible?

b. Choledochal cyst Obstructive jaundice (ie, direct-reacting bilirubin greater than 20% of the total) requires investigation in all infants. CF and α1-antitrypsin deficiency should be considered in the diagnostic evaluation of any child with obstructive jaundice.

438. A 7-day-old boy is admitted to a hospital for evaluation of vomiting and dehydration. Physical examination is otherwise normal except for minimal hyperpigmentation of the nipples. Serum sodium and potassium concentrations are 120 mEq/L and 9 mEq/L (without hemolysis), respectively; serum glucose is 40 mg/dL. Which of the following is the most likely diagnosis?

b. Congenital adrenal hyperplasia Salt-losing congenital adrenal hyperplasia (adrenogenital syndrome, 21-hydroxylase deficiency) usually manifests during the first 5 to 15 days of life as anorexia, vomiting, diarrhea, and dehydration. Hypoglycemia can also occur. In classic 21-hydroxylase deficiency, serum levels of 17-hydroxyprogesterone are markedly elevated beyond 3 days of life (in the first 3 days of life they can normally be high). Blood cortisol levels are usually low in salt-losing forms of the disease.

150. A previously healthy full-term infant has several episodes of duskiness and apnea during the second day of life. What congenital disease must be ruled out?

b. Congenital heart disease Idiopathic apnea is common in premature infants but is not expected in the full- term newborn. When apnea occurs in the term infant, there is almost always an identifiable cause. Sepsis, gastroesophageal reflux, congenital heart disease, seizures, RSV, hypoglycemia, central hypoventilation (Ondine's curse), and airway obstruction can all cause apnea in term newborns.

33. You are seeing an established patient, a 4-year-old girl brought in by her mother for vaginal itching and irritation. She is toilet trained and has not complained of frequency or urgency, nor has she noted any blood in her urine. Her mother noted she has been afebrile and has not complained of abdominal pain. Mom denies the risk of inappropriate contact; the girl also denies anyone "touching her there." Your physical examination of the perineum is significant for the lack of foul odor or discharge. You do note some erythema of the vulvar area but no evidence of trauma. Which of the following is the most appropriate course of action?

b. Counsel mother to stop giving the girl bubble baths, have the girl wear only cotton underwear, and improve hygiene.

65. A 6-week-old boy was small for his birth weight and had intrauterine growth retardation. He is microcephalic, has a rounded face, hypertelorism, and epicanthal folds. His cry is high-pitched

b. Cri du chat syndrome

361. The parents of a 7-day-old infant bring her to your office for a swollen eye. Her temperature has been normal, but for the last 2 days she has had progressive erythema and swelling over the medial aspect of the right lower lid near the punctum. Her sclera and conjunctiva are clear. Gentle pressure extrudes a whitish material from the punctum. Which of the following ophthalmic conditions is the correct diagnosis?

b. Dacryocystitis Dacryocystitis is an infection of the nasolacrimal sac. In newborns it is associated with congenital nasolacrimal duct obstruction, which is seen in about 6% of normal infants. Nasolacrimal duct obstruction is thought to be caused by the failure of epithelial cells forming the duct to canalize.

Questions 76 to 78 76. A 14-year-old male presents after taking a "happy pill" that his friend gave him. He is alert and oriented, but complains of a muscle spasm in his neck, making his head lean on his right shoulder. You also notice he is arching his back in an unusual manner.

b. Diphenhydramine (Benadryl) Phenothiazine can cause an idiosyncratic reaction causing extrapyramidal symptoms such as oculogyric crisis, tremors, torticollis, opisthotonus, and dysphagia. These dystonic symptoms respond surprisingly quickly to the intravenous or intramuscular administration of diphenhydramine (Benadryl).

491. The parents of a 16-year-old girl complain that she does not get enough sleep. They recently discovered that she stays awake most nights until 1:00 AM reading and text messaging her friends. She wakes at 6:30 AM for school, and complains of sleepiness during the day. On weekends she sleeps until noon. Her parents have tried taking away her computer and phone, but she still would go to bed at the same time. The parents are looking for advice in dealing with their "night owl" daughter. Which of the following is appropriate advice for this family?

b. Effects of puberty on melatonin cause a phase delay with later sleep onset.

142. The mother and father of a newborn come in for the 2-week checkup. The mother complains of "colic" and asks if she can switch to goat's milk instead of breast milk. What should be your main concern about using goat's milk instead of breast milk or cow's milk?

b. It has insufficient folate. Goat's milk, by itself, is not an ideal source of infant nutrition as it contains inadequate folate and iron, potentially contributing to anemia. It also is usually unpasteurized.

262. An 18-month-old infant is found with the contents of a bottle of drain cleaner in his mouth. Which of the following treatment options is most appropriate?

b. Endoscopic examination within the first 12 to 24 hours Endoscopic examination of the esophagus and stomach is a diagnostic method of determining the extent of the mucosal injury. Vomiting is to be avoided since it would expose the mucosal surfaces to the caustic agent a second time. The child can be given small amounts of milk or water, but large amounts, which might cause vomiting, are unwise. Neutralization of the caustic liquid can result in an exothermic reaction and produce a thermal burn. Charcoal, however, does not absorb the alkaline agent in drain cleaner. Esophageal strictures are the primary long-term complication of caustic ingestion.

10. A chubby 6-month-old baby boy is brought to the clinic by his father. His father is concerned that his penis is too small (see photograph). The child is at the 95% for weight and the 50% for length; he has been developing normally and has had no medical problems. Which of the following is the most appropriate first step in management of this child?

b. Evaluation of penile length after retracting the skin and fat lateral to the penile shaft

148. A well-appearing, 3200-g (7-lb, 1-oz) black infant is noted to have fifth finger (postaxial) polydactyly. The extra digit has no skeletal duplications and is attached to the rest of the hand by a threadlike soft tissue pedicle (see photograph). Appropriate treatment for this condition includes which of the following?

b. Excision of extra digit This finding in otherwise healthy black children should raise no special concern. In a white child, careful examination of the cardiac system is warranted. Trisomy 13 is associated with polydactyly

266. A 2-year-old has crampy abdominal pain and grossly bloody diarrheal stool, but no fever. His abdominal examination reveals no masses. A family member, who ate at the same local hamburger shop the night prior, has the same symptoms.

b. Fiberoptic endoscopy

268. A 10-year-old has complained for 1 month of intermittent epigastric pain that awakens him from sleep. He notes that eating food sometimes helps. He reports black stools during the prior week, and also admits that he has occasionally vomited frank blood.

b. Fiberoptic endoscopy

58. A skier recently rescued from a snowbank following an avalanche (caused by a barking Chihuahua) complains about his feet. Upon rescue they were whitish yellow and numb, but now they are blotchy and painful.

b. Frostbite

281. A 17-year-old boy is brought to the emergency department by his parents with the complaint of coughing up blood. He is stabilized, and his hemoglobin and hematocrit levels are 11 mg/dL and 33%, respectively. During his hospitalization, he is noted to have systolic blood pressure persistently greater than 130 mm Hg and diastolic blood pressure greater than 90 mm Hg. His urinalysis is remarkable for hematuria and proteinuria. You are suspicious the patient has which of the following?

b. Goodpasture syndrome The patient in the question has a classic description of Goodpasture disease, a rare disease in children. The pulmonary hemorrhage can be life threatening and the renal impairment progressive. Diagnosis is suggested by finding hemosiderin-laden macrophages on sputum or pulmonary secretions and the finding of antibodies to the glomerular basement membrane; a kidney biopsy may be required as well.

288. A 4-year-old boy, whose past medical history is positive for three urinary tract infections, presents with a blood pressure of 135/90 mm Hg. He is likely to exhibit which of the following symptoms or signs?

b. Headache Routine BP measurement is performed on children of age 3 and beyond and for subgroups of children at younger ages, including children who were born prematurely, had frequent UTI, had umbilical artery catheters, and so on. Once a baby suspected to have hydronephrosis or another urinary system abnormality is born, the baby's blood pressure will be monitored carefully, because some kidney problems can cause high blood pressure.

418. An infant is born to a woman who has received very little prenatal care. The mother is anxious, complains of heat intolerance and fatigue, and reports that she has not gained much weight despite having an increased appetite. On examination the mother is tachycardic, has a tremor, and has fullness in her neck and in her eyes. The infant is most likely at risk for development of which of the following?

b. Heart failure The infant is likely at risk for neonatal thyrotoxicosis. Neonatal thyrotoxicosis usually disappears within 2 to 4 months as the concentration of maternally acquired thyrotropin receptor-stimulating antibody (TRSAb) falls. Symptoms include tachycardia and tachypnea, irritability and hyperactivity, low birth weight with microcephaly, severe vomiting and diarrhea, thrombocytopenia, jaundice, hepatosplenomegaly, and heart failure. In severely affected infants, the disease can be fatal if not treated vigorously and promptly. Third-degree heart block is not a feature of this disease, but is sometimes seen in infants born to mothers with systemic lupus erythematosus.

310. A 6-month-old child was noted to be normal at birth, but over the ensuing months you have been somewhat concerned about his slowish weight gain and his mild delay in achieving developmental milestones. The family calls you urgently at 7:00 AM noting that their child seems unable to move the right side of his body. Which of the following conditions might explain this child's condition?

b. Homocystinuria Manifestations include poor growth, arachnodactyly, osteoporosis, dislocated lenses, and mental retardation. In addition, *thromboembolic* phenomena may be seen in the pulmonary and systemic arteries and particularly in the cerebral vasculature; vascular occlusive disease is, in turn, one of the many causes of acute infantile hemiplegia.

251. A 15-year-old otherwise healthy boy presents with a complaint of intermittent abdominal distention, crampy abdominal pain, and excessive flatulence. He first started noticing these symptoms when he moved into his father's house, and his stepmother insisted on milk at dinner every night. He has normal growth, has not lost weight, and has no travel history. Which of the following is the most appropriate study to diagnose his condition?

b. Hydrogen excretion in breath after oral administration of lactose The clinical symptoms of lactose malabsorption are due to the presence of osmotically active, undigested lactose, which may act to increase intestinal fluid volume, alter transit time, and produce the symptoms of abdominal cramps, distention, and occasionally, watery diarrhea. Bacterial metabolism of the nonabsorbed carbohydrates in the colon into carbon dioxide and hydrogen may contribute to the clinical symptoms. Acquired lactase deficiency is often associated with conditions of the GI tract that cause intestinal mucosal injury (eg, sprue and regional enteritis). Diagnostic techniques for lactose intolerance include removal of the offending sugar, with a reproduction of symptoms upon reintroduction.

Questions 89 to 92 89. An 18-year-old friend of the family returns from spring break from a coastal town in Central America. He has an intensely pruritic lesion on his foot. The lesion is raised, red, serpiginous, and has a few associated bullae.

b. Ivermectin Cutaneous larva migrans (also known as creeping eruption) is caused primarily by Ancylostoma braziliense, a dog and cat hookworm. After exposure (such as walking barefoot on a beach and stepping where an infected dog has recently been), the larvae penetrate the skin at the epidermal-dermal junction and migrate at about 1 to 2 cm a day. The result is an intensely pruritic lesion as described in the vignette. Left untreated the larvae die over a period of weeks to months, but treatment with antihelminthic medications hastens resolution of symptoms.

189. A child has a 2-week history of spiking fevers, which have been as high as 40°C (104°F). She has spindle-shaped swelling of finger joints and complains of upper sternal pain. When she has fever, the parents note a faint salmon-colored rash that resolves with the resolution of the fever. She has had no conjunctivitis or mucositis, but her heart sounds are muffled and she has increased pulsus paradoxus. Which of the following is the most likely diagnosis?

b. Juvenile rheumatoid arthritis Presentation of JIA occurs as either polyarthritis (five or more joints, systemic symptoms not so severe or persistent), pauciarticular (four or fewer joints, lower-extremity joints, extra-articular disease unusual), or systemic disease (severe constitutional disease, systemic symptoms prior to arthritis, rheumatoid rash, high spiking fevers, variable joint involvement, also known as Still's disease).

457. An obese 14-year-old boy with hypogonadism and night blindness with retinitis pigmentosa

b. Laurence-Moon-Biedl syndrome Laurence-Moon-Biedl (Bardet-Biedl) syndrome is transmitted as an autosomal recessive trait. Obesity, mental retardation, hypogonadism, polydactyly, and retinitis pigmentosa with night blindness are the principal findings in affected children. There is no known effective treatment.

32. During the examination of a 2-month-old infant, you note that the infant's umbilical cord is still firmly attached. This finding prompts you to suspect which of the following?

b. Leukocyte adhesion deficiency

475. A 15-year-old female presents to your office with secondary amenorrhea. As part of your evaluation, you find that she is pregnant. After informing her of the pregnancy, you continue to explain that young mothers have a higher risk of several pregnancy-related complications, including which of the following?

b. Low-birthweight infants Adolescents are typically healthy and do not have chronic disease. However, several pregnancy complications occur more frequently in teens, including poor weight gain in the mother, premature delivery, low birth weight, and an increased risk of pregnancy-induced hypertension. The risk of violence is also elevated for teen mothers.

214. A 10-year-old girl has had a "cold" for 14 days. In the 2 days prior to the visit to your office, she has developed a fever of 39°C (102.2°F), purulent nasal discharge, facial pain, and a daytime cough. Examination of the nose after topical decongestants shows pus in the middle meatus. Which of the following is the most likely diagnosis?

b. Maxillary sinusitis Maxillary and ethmoid sinuses are large enough to harbor infections from infancy. Frontal sinuses are rarely large enough to harbor infections until the sixth to tenth year of life. Sphenoid sinuses do not become large until about the third to fifth year of life. In general, a "cold" lasting longer than 10 to 14 days with fever and facial pain is suggestive of rhinosinusitis.

23. You are seeing a 2-year-old child, brought by his father for a well-child examination. In providing age-appropriate anticipatory guidance, you should tell him which of the following?

b. Milk should be switched from whole to skim or low fat.

72. 16-year-old girl took "some pills." Complains of emesis, diaphoresis, and malaise. Her initial liver function tests, obtained about 12-hour postingestion, are elevated. Repeat levels at 24-hours show markedly elevated aspartate aminotransferase (AST) and alanine aminotransferase (ALT), along with abnormal coagulation studies and an elevated bilirubin.

b. N-acetylcysteine (Mucomyst) N-acetylcysteine (NAC) is an effective treatment for acetaminophen poisoning and acts as a glutathione substitute by binding directly to N-acetyl- p-benzoquinone imine, the cytotoxic metabolite of acetaminophen.

463. A 1-year-old girl with glucose-6-phosphatase deficiency (von Gierke disease)

b. Na+ 125, K+ 3.0 Patients with a deficiency of glucose-6-phosphatase (von Gierke disease) are, as a rule, hyperlipidemic. Increased triglyceride concentration in the serum increases the volume of the aqueous compartment. Because electrolytes are present only in the aqueous compartment of the serum but are expressed in milliequivalents per liter of serum as a whole, the concentrations of sodium and potassium can be factitiously low in these patients.

18. The mother of a 3-day-old infant brings her child to your office for an early follow-up visit. The mom notes that the child has been eating well, has had no temperature instability, and stools and urinates well. She notes that over the previous 3 days the child has had a progressive "rash" on the face as pictured here. Which of the following is the most likely diagnosis?

b. Neonatal acne

52. A 4-year-old boy occasionally wakes in the middle of the night crying. When his parents check on him, he seems visibly frightened and tells his parents that Chihuahuas were chasing him.

b. Nightmares

394. An 11-month-old African American boy has a hematocrit of 24% on a screening laboratory done at his well-child checkup. Further testing demonstrates: hemoglobin 7.8 g/dL; hematocrit 22.9%; leukocyte count 12,200/μL with 39% neutrophils, 6% bands, 55% lymphocytes; hypochromia on smear; free erythrocyte protoporphyrin (FEP) 114 μg/dL; lead level 6 μg/dL whole blood; platelet count 175,000/μL; reticulocyte count 0.2%; sickle-cell preparation negative; stool guaiac-negative; and mean corpuscular volume (MCV) 64 fL. Which of the following is the most appropriate recommendation?

b. Oral ferrous sulfate Response to a therapeutic trial of iron is an appropriate and cost-effective method of diagnosing iron-deficiency anemia. A prompt reticulocytosis and rise in hemoglobin and hematocrit follow the administration of an oral preparation of ferrous sulfate.

Questions 54 to 58 54. A 6-year-old returns from playing all day in the snow with several erythematous, ulcerative lesions on his fingertips; he complains the lesions are painful and itchy.

c. Chilblain

25. You are called by a general practitioner to consult on a patient admitted to the hospital 4 days ago. The patient is a 7-month-old white boy with poor weight gain for the past 3 months, who has not gained weight in the hospital despite seemingly adequate nutrition. You take a detailed diet history from his foster mother, and the amounts of formula and baby food intake seem appropriate for age. Physical examination reveals an active, alert infant with a strong suck reflex who appears wasted. You note generalized lymphadenopathy with hepatomegaly. In addition, you find a severe case of oral candidiasis that apparently has been resistant to treatment. Which of the following is the most appropriate next step in the evaluation or treatment of this child?

b. Order human immunodeficiency virus (HIV) polymerase chain reaction (PCR). testing because this is likely the presentation of congenitally acquired HIV.

477. A 16-year-old boy who is the backup quarterback for the local high school team is in your office complaining of worsening acne. For the last few months he has noted more acne and more oily hair. On his examination, you note gynecomastia and small testicular volume. He is SMR 5. Which of the following drugs of abuse is the likely explanation for all of his findings?

b. Oxandrolone an anabolic steroid

37. Aunt Mary is helping her family move to a new apartment. During the confusion, 3-year-old Jimmy is noted to become lethargic. The contents of Aunt Mary's purse are strewn about on the floor. In the ER, the lethargic Jimmy is found to have miosis, bradycardia, and hypotension. He develops apnea, respiratory depression, and has to be intubated. His condition would most likely benefit from which of the following therapies?

b. Pediatric intensive care unit (PICU) support and trial of naloxone

19. A 2-year-old child (A) presents with a 4-day history of a rash limited to the feet and ankles. The papular rash is both pruritic and erythematous. The 3-month-old sibling of this patient (B) has similar lesions also involving the head and neck. The most appropriate treatment for this condition includes which of the following?

b. Permethrin

140. 4 day old boy. The parents of the child are concerned because the pediatrician noted the child was "yellow" and ordered some studies. They produce a wad of papers for you to review. Both the mother and baby have O-positive blood. The baby's direct serum bilirubin is 0.2 mg/dL, with a total serum bilirubin of 11.8 mg/dL. Urine bilirubin is positive. The infant's white blood cell count is 13,000/μL with a differential of 50% polymorphonuclear cells, 45% lymphocytes, and 5% monocytes. The hemoglobin is 17 g/dL, and the platelet count is 278,000/μL. Reticulocyte count is 1.5%. The peripheral smear does not show fragments or abnormal cell shapes. Which of the following is the most likely explanation for this infant's skin color?

b. Physiologic jaundice WBC should be high in the newborn.

9. The adolescent shown presents with a 14-day history of multiple oval lesions over her back. The rash began with a single lesion over the lower abdomen (A); the other lesions developed over the next days (B). These lesions are slightly pruritic. Which of the following is the most likely diagnosis?

b. Pityriasis rosea

392. An 8-year-old girl with fever, tachypnea, and lobar infiltrate. She has failed outpatient therapy of amoxicillin, has developed empyema, and has had to have chest tubes placed.

b. Pneumococcal pneumonia Pneumococcal pneumonia often presents with sudden onset of fever, cough, and chest pain. This child may have failed outpatient therapy because of increased incidence of *resistant organisms*. Pleural effusions or empyema are commonly seen; chest tube evacuation of the fluid is often required. Depending on sensitivity patterns in a particular community, therapy with high-dose penicillin, cefuroxime, amoxicillin/clavulanate, or even vancomycin may be required.

345. A 2-month-old child of an HIV-positive mother is followed in your pediatric practice. Which of the following therapies should be considered for this child?

b. Prophylaxis against Pneumocystis jiroveci pneumonia (Pneumocystis carinii)

381. Foot puncture wound through a tennis shoe of an adolescent exploring a construction site

b. Pseudomonas Pseudomonas is a ubiquitous organism. Most infections with the organism are opportunistic, involving several organs. Skin infections related to burns, trauma (such as in a puncture wound through a tennis shoe), and use of swimming pools are not uncommon. Injuries through a tennis shoe are prone to pseudomonal infections because of the warm, moist nature of the shoe's environment. In contrast, a wound through a bare foot would be associated with cutaneous flora such as Staphylococcus.

Questions 96 to 102 96. Isoniazid therapy in a pregnant teenager What supplement is indicated for baby?

b. Pyridoxine

302. A 21-year-old woman with no prenatal care, presents to hospital in active labor. Her membranes are artificially ruptured, yielding no amniotic fluid. She delivers an 1800-g (4-lb) term infant who develops significant respiratory distress immediately at birth. The first chest radiograph on this infant demonstrates hypoplastic lungs. After this infant is stabilized, which of the following is the most appropriate next step for this infant?

b. Renal ultrasound Oligohydramnios can cause a number of serious problems in the infant, including constraint deformities (such as clubfoot) and pulmonary hypoplasia, as shown in the photograph which shows essentially no lung development. These infants have usually experienced intrauterine growth retardation and frequently have an associated serious renal abnormality. Ultrasound of the kidneys is important to rule out renal involvement as a cause of the oligohydramnios. The finding of bilateral renal agenesis is termed Potter sequence.

476. A 12-year-old boy has scant, long, slightly pigmented pubic hairs; slight enlargement of his penis, and a pink, textured, and enlarged scrotum. He is most likely at which sexual maturation rating (SMR, also called Tanner) stage?

b. SMR 2

44. An adolescent boy complains of a splotchy red rash on the nape of his neck, discovered when he had his head shaved for football season. The rash seems to become more prominent with exercise or emotion. His mother notes that he has had the rash since infancy, but that it became invisible as hair grew. He had a similar rash on his eyelids that resolved in the newborn period.

b. Salmon patch

53. A 5-year-old child refuses to sleep in his bed, claiming there are monsters in his closet and that he has bad dreams. The parents allow him to sleep with them in their bed to avoid the otherwise inevitable screaming fit. The parents note that the child sleeps soundly, waking only at sunrise.

c. Learned behavior

171. A new mother points out several hypopigmented oval macules over the child's trunk and extremities. She notes that these have been present since birth. She is concerned because she had a brother with the same spots whom she thinks had "growths" in his brain. What is the diagnosis?

b. Seizures Infants who have tuberous sclerosis are often born with hypopigmented oval or irregularly shaped skin macules (ash leaf spots). Cerebral sclerotic tubers also present from birth and become visible radiographically by the third to fourth year of life. Myoclonic seizures, present in infancy, can convert to grand mal seizures later in childhood. Adenoma sebaceum appears at 4 to 7 years of age. The disease, which also affects the eyes, kidneys, heart, bones, and lungs, is inherited as an autosomal dominant trait with variable expression; new mutations are very common.

446. A 1-day-old normal-appearing infant develops tetany and convulsions. He was born at 34 weeks' gestation with Apgar scores of 2 and 4 (at 1 and 5 minutes, respectively) to a woman whose pregnancy was complicated by diabetes mellitus and pregnancy-induced hypertension. Which of the following serum chemistry values is likely to be the explanation for his condition?

b. Serum calcium of 6.2 mg/dL Hypocalcemia of newborn infants can be divided into two groups: early (during the first approximately 72 hours of life) and late (after approximately 72 hours). The most common type of early neonatal hypocalcemia is the so-called idiopathic hypocalcemia.

Questions 67 to 70 67. An afebrile, obese 14-year-old boy has developed pain at the right knee and a limp.

b. Slipped capital femoral epiphysis...

495. A 19-year-old female has a few small papules on her labia and perineum. The papules become pustular, eroded, and ulcerated over the next few days; at the same time, the patient develops painful, tender inguinal lymphadenopathy.

b. Special chocolate agar culture

5. A previously healthy 4-year-old child pictured below presents to the emergency room (ER) with a 2-day history of a brightly erythematous rash and temperature of 40°C (104°F). The exquisitely tender, generalized rash is worse in the flexural and perioral areas. The child is admitted and over the next day develops crusting and fissuring around the eyes, mouth, and nose. The desquamation of skin shown in the photograph occurs with gentle traction. Which of the following is the most likely diagnosis?

b. Staphylococcal scalded skin syndrome

153. Parents bring a 5-day-old infant to your office. The mother is O negative and was Coombs positive at delivery. The term child weighed 3055 g (6 lb, 1 oz) at birth and had measured baseline hemoglobin of 16 g/dL and a total serum bilirubin of 3 mg/dL. He passed a black tarlike stool within the first 24 hours of life. He was discharged at 30 hours of life with a stable axillary temperature of 36.5°C (97.7°F). Today the infant's weight is 3000 g, his axillary temperature is 35°C (95°F), and he is jaundiced to the chest. Parents report frequent yellow, seedy stool. You redraw labs and find his hemoglobin is now 14 g/dL, and his total serum bilirubin is 13 mg/dL. The change in which of the following parameters is of most concern?

b. Temperature (SEPSIS!)

331. A newborn infant has respiratory distress and trouble feeding in the nursery. The mother has no significant medical history, but the pregnancy was complicated by decreased fetal movement. On physical examination, you note that aside from shallow respirations and some twitching of the fingers and toes, the infant is not moving, and is very hypotonic. In the mouth there is pooled saliva and you note tongue fasciculations. Deep tendon reflexes are absent. Spinal fluid is normal. Appropriate statements about this condition include which of the following statements? Diagnosis?

b. The condition is caused by the degeneration of anterior horn cells in the spinal cord. The infant described has spinal muscular atrophy (SMA) type I, also referred to as Werdnig-Hoffman disease, or infantile progressive SMA. The defect is found in the survivor motor neuron (SMN) gene that stops apoptosis of motor neuroblasts. During development, an excess of motor neuroblasts is noted, and through apoptosis only about half survive in the normal newborn; the SMN gene regulates this natural destruction. A defect in the SMN gene results in a continuation of apoptosis, resulting in progressive destruction of motor neurons in the brain stem and spinal cord. The only currently available treatment is supportive care, and infants with SMA I usually die of respiratory complications by the second or third year of life.

191. A 3-day-old infant with a single second heart sound has had progressively deepening cyanosis since birth but no respiratory distress. Chest radiography demonstrates no cardiomegaly and normal pulmonary vasculature. An ECG shows an axis of 120° and right ventricular prominence. Which of the following congenital cardiac malformations is most likely responsible for the cyanosis?

b. Transposition of the great vessels Transposition of the great vessels with an intact ventricular septum presents with early cyanosis, a normal-sized heart (classic "egg on a string" radiographic pattern in one-third of cases), normal or slightly increased pulmonary vascular markings, and an ECG showing right-axis deviation and right ventricular hypertrophy. In tetralogy of Fallot, cyanosis is often not seen in the first few days of life. Tricuspid atresia, a cause of early cyanosis, causes diminished pulmonary arterial blood flow; the pulmonary fields on x-ray demonstrate a diminution of pulmonary vascularity, and left-axis and LVH are shown by ECG. Total anomalous pulmonary venous return below the diaphragm is associated with obstruction to pulmonary venous return and a classic radiographic finding of marked, fluffy-appearing venous congestion ("snowman"). In pulmonic atresia with an intact ventricular septum, cyanosis appears early, the lung markings are normal to diminished, and the heart is large.

469. A small newborn with a large ventricular septal defect (VSD), *clenched hands*, cleft palate, rounded heels, and a horseshoe kidney.

b. Trisomy 18 (Edwards syndrome)

318. Examination of the cerebrospinal fluid (CSF) of an 8-year-old, mildly febrile child with nuchal rigidity and intermittent stupor shows the following: WBCs 85/μL (all lymphocytes), negative Gram stain, protein 150 mg/dL, and glucose 15 mg/dL. A computed tomographic (CT) scan with contrast shows enhancement of the basal cisterns by the contrast material. Which of the following is the most likely diagnosis?

b. Tuberculous meningitis Viral meningitis, tuberculous meningitis, meningeal leukemia, and medulloblastoma, all of which can cause pleocytosis as well as elevated protein and lowered glucose concentrations in CSF, Of the four diseases, tuberculous meningitis is associated with the lowest glucose levels in CSF. The CT scan with contrast can be an excellent clue for diagnosing tuberculous meningitis. Exudate in the basal cisterns that shows enhancement by contrast material is typical; tuberculomas, ringed lucencies, edema, and infarction can be apparent; and hydrocephalus can develop.

The x-ray of a 60year old girl reveals a lung empyema on the left side, characterized by nearly complete white out of that side by the pleural or extrapleural fluid collection. What is the treatment? What are the most likely pathogens?

b. Vancomycin Lung abscesses are usually caused by S aureus, S pneumoniae, or S pyogenes. These organisms, previously sensitive to penicillin, now have a variety of resistance patterns requiring extended spectrum antibiotics or vancomycin.

Questions 269 to 274 269. A 15-year-old vegetarian being treated for tuberculosis develops peripheral neuropathy

b. Vitamin B6

402. While bathing her newly-received 2-year-old son, a foster mother feels a mass in his abdomen. A thorough medical evaluation of the child reveals aniridia, hypospadias, horseshoe kidney, and hemihypertrophy. Which of the following is the most likely diagnosis for this child?

b. Wilms tumor

396. On a routine-screening CBC, a 1-year-old is noted to have a microcytic anemia. A follow-up hemoglobin electrophoresis demonstrates an increased concentration of hemoglobin A2. The child is most likely to have which of the following?

b. β-Thalassemia trait

254. An awake, alert infant with a 2-day history of diarrhea presents with a depressed fontanelle, tachycardia, sunken eyes, and the loss of skin elasticity. Which of the following is the correct percentage of dehydration?

c. 5% to 9% Additional findings at this level of dehydration can be restlessness, absent or reduced tears, tachycardia with weak radial pulses, decreased urine output, cool skin, and possibly orthostatic hypotension. *not severe because there is no hyponatremia

132. A term infant is born to a known HIV-positive mother. She has been taking antiretroviral medications for the weeks prior to the delivery of her infant. Routine management of the healthy infant should include which of the following?

c. A course of zidovudine for the infant

296. The mother of a 2-year-old girl reports that her daughter complains of burning when she urinates and that she has foul-smelling discharge from her vagina. She has some slight staining on the front of her underwear, but denies fever, nausea, vomiting, or other constitutional signs. The child does not attend day care, and she has demonstrated no change in behavior. The physical examination is normal with an intact hymen, but the child's vulva is reddened and with a malodorous scent noted. Her urinalysis and culture are normal. Management of this condition includes which of the following?

c. Advice to stop taking prolonged bubble baths The symptoms listed are those of vulvovaginitis, with nonspecific (or chemical) vulvovaginitis accounting for 70% of all pediatric vulvovaginitis cases. The discharge in nonspecific vulvovaginitis is usually brown or green and with a fetid odor. The burning with urination occurs because of contact between raw skin and urine.

64. A 17-year-old boy has an unusual gait, large mouth with tongue protrusion, hypopigmentation with blond hair and pale blue eyes, and unprovoked bursts of laughter

c. Angelman syndrome

Questions 264 to 268 264. A newborn infant, the product of an emergency cesarean section, is 24 hours old and has a grossly bloody stool. She looks well otherwise.

c. Apt test

306. A 12-month-old girl whose height and weight are less than the fifth percentile; she has had several bouts of constipation and two previous admissions for dehydration. She is again admitted for dehydration and is noted to have serum potassium of 2.7 mEq/L.

c. Bartter syndrome Bartter syndrome (also known as juxtaglomerular hyperplasia) is an autosomal recessive condition that causes hypokalemia, hypercalciuria, alkalosis, hyperaldosteronism, and hyperreninemia; blood pressure is usually normal. Clinical presentations occurring frequently between 6 and 12 months of age include failure to thrive with constipation, weakness, vomiting, polyuria, and polydipsia. Treatment is aimed at preventing dehydration, providing nutritional support, and returning the potassium level to normal.

Questions 84 to 88 84. A 15-year-old with severe acne on an oral preparation of retinoic acid.

c. Bilateral microtia or anotia Isotretinoin is a very effective medication for nodular cystic acne, but if taken when a woman is pregnant commonly results in teratogenic effects including hydrocephalus, CNS defects, microtia/anotia, small or missing thymus, conotruncal heart defect, micrognathia, and even fetal death.

186. A 3-month-old infant is brought to your office for pallor and listlessness. Your physical examination reveals tachycardia that is constant and does not vary with crying. He has no hepatomegaly and the lungs are clear. His ECG is shown. Which of the following is the most appropriate initial management of this patient?

c. Carotid massage

A 6-week-old infant with tachypnea and history of eye discharge at 2 weeks of age. Diagnosis?

c. Chlamydial pneumonia Approximately 10% to 20% of infants born to mothers with Chlamydia trachomatis infection develop pneumonia. The presentation of this pneumonia usually occurs between 1 and 3 months of age, with cough, *tachypnea*, and lack of fever. Examination reveals rales but not wheezing. Laboratory data suggestive of C trachomatis infection include an increase in eosinophils in the peripheral blood.

488. An 18-year-old male college student is seen in the student health clinic for urinary frequency, dysuria, and urethral discharge. Which of the following is likely to explain his condition?

c. Chlamydial urethritis Urethritis in an adolescent male is almost always a sexually transmitted disease (STD), either gonococcal or nongonococcal urethritis (NGU). Chlamydia trachomatis is usually the causative agent in NGU

436. A 15-year-old boy has been immobilized in a double hip spica cast for 6 weeks after having fractured his femur in a skiing accident. He has become depressed and listless during the past few days and has complained of nausea and constipation. He is found to have microscopic hematuria and a blood pressure of 150/100 mm Hg. Which of the following is the most appropriate course of action?

c. Collect urine for measurement of the calcium to creatinine ratio Hypercalcemia can develop in children who are immobilized following the fracture of a weight-bearing bone. Serious complications of immobilization hypercalcemia, and the hypercalciuria that occurs as a result, include nephropathy, nephrocalcinosis, hypertensive encephalopathy, and convulsions. The early symptoms of hypercalcemia—namely, constipation, anorexia, occasional vomiting, polyuria, and lethargy—are nonspecific and may be ascribed to the effects of the injury and hospitalization. Therefore, careful monitoring of these patients with serial measurements of the serum-ionized calcium and the urinary calcium to creatinine ratio is critical during their immobilization.

Questions 156 to 159 156. A newborn has bilateral cataracts and microphthalmia, intrauterine growth retardation, hemorrhagic skin lesions scattered throughout the body, and a harsh systolic murmur heard at the left sternal border and radiating to the lung fields.

c. Congenital rubella

36. A 5-year-old boy is brought into the ER immediately after an unfortunate altercation with a neighbor's immunized Chihuahua that occurred while the child was attempting to dress the dog as a superhero. The fully immunized child has a small, irregular, superficial laceration on his right forearm that has stopped bleeding. His neuromuscular examination is completely normal, and his perfusion is intact. Management should include which of the following?

c. Copious irrigation

247. A 14-year-old girl has a 9-month history of diarrhea, abdominal pain (usually periumbilical and postprandial), fever, and weight loss. She has had several episodes of blood in her stools. Which of the following is the most likely diagnosis?

c. Crohn disease The presentation of Crohn disease (granulomatous colitis) depends on the location and extent of lesions. Onset of the GI or extraintestinal symptoms can be insidious. The "textbook" presentation is as described, although only 25% of patients have the "triad" of diarrhea, weight loss, and abdominal pain. Crohn disease characteristically is associated with transmural, granulomatous intestinal lesions that are discontinuous and can appear in both the small and large intestines. Although Crohn disease can first appear as a rectal fissure or fistula, the rectum is often spared. Arthritis/arthralgia occurs in a minority of affected children. Other extraintestinal symptoms include erythema nodosum or pyoderma gangrenosum, liver disease, renal calculi, uveitis, anemia, specific nutrient deficiency, and growth failure. In relation to the general population, the risk of colonic carcinoma in affected persons is increased, but not to the degree associated with ulcerative colitis.

282. A 1-year-old child presents with failure to thrive, frequent large voids of dilute urine, excessive thirst, and three episodes of dehydration not associated with vomiting or diarrhea. Over the years, other family members reportedly have had similar histories. Which of the following is the most likely diagnosis?

c. Diabetes insipidus Dx: Repeated measurement of serum sodium and urine specific gravity levels during a controlled fluid restriction challenge The case is that of a child with classic central nephrogenic diabetes insipidus, a hereditary disorder in which the urine is hypotonic and produced in large volumes because the distal tubule and collecting duct of the kidneys fail to respond to antidiuretic hormone (vasopressin). Males are primarily affected through an X-linked recessive inheritance; autosomal dominant and recessive forms are also known.

Questions 71 to 75 71. Over the past several weeks, a 2-year-old girl has exhibited developmental regression, abnormal sleep patterns, anorexia, irritability, and decreased activity. These symptoms have progressed to acute encephalopathy with vomiting, ataxia, and variable consciousness. The family recently moved, and they are in the process of restoring the interior of their home.

c. Dimercaptosuccinic acid (DMSA, succimer) Doctors treat adults with lead levels greater than 45 mcg/dL of blood and children who can't tolerate the drug used in conventional chelation therapy most commonly with a chemical called calcium disodium ethylenediaminetetraacetic acid (EDTA). EDTA is given by injection.

407. A 10-year-old boy is admitted to the hospital because of bleeding. Pertinent laboratory findings include a platelet count of 50,000/μL, prothrombin time (PT) of 15 seconds (control 11.5 seconds), activated partial thromboplastin time (aPTT) of 51 seconds (control 36 seconds), thrombin time (TT) of 13.7 seconds (control 10.5 seconds), and factor VIII level of 14% (normal 38%-178%). Which of the following is the most likely cause of his bleeding?

c. Disseminated intravascular coagulation (DIC) In DIC, there is consumption of fibrinogen; factors II, V, and VIII; and platelets. Therefore, there is prolongation of PT, aPTT, and TT and a decrease in factor VIII level and platelet count. In addition, the titer of fibrin split production is usually increased. D-dimer is a fibrin breakdown product and may also be elevated in DIC.

183. A 2-year-old boy is brought into the emergency room with a complaint of fever for 6 days and the development of a limp. On examination, he is found to have an erythematous macular exanthem over his body as shown in image A, ocular conjunctivitis, dry and cracked lips, a red throat, and cervical lymphadenopathy. There is a grade 2/6 vibratory systolic ejection murmur at the lower left sternal border. A white blood cell (WBC) count and differential show predominant neutrophils with increased platelets on smear. Later, he develops the findings as seen in image B. Which of the following is the most likely diagnosis?

c. Kawasaki disease

213. A 4-year-old boy was admitted to the hospital last night with the complaint of "difficulty breathing." He has no past history of lung infection, no recent travel, and no day-care exposure; he does, however, have an annoying tendency to eat dirt. In the emergency center he was noted to be wheezing and to have hepatomegaly. He is able to talk, relaying his concern about his 6-week-old Chihuahua being left alone at home. Laboratory studies revealed marked eosinophilia (60% eosinophils). Which of the following tests is most likely to produce a specific diagnosis?

c. ELISA for Toxocara The presentation described is characteristic of visceral larva migrans from infestation with a common parasite of dogs, Toxocara canis. Dirt-eating children ingest the infectious ova. The larvae penetrate the intestine and migrate to visceral sites, such as the liver, lung, and brain, but do not return to the intestine, so the stools do not contain the ova or parasites. The diagnosis can be made by a specific ELISA for Toxocara. Patients with minimal symptoms may be managed expectantly, while patients with more significant symptoms may be treated with albendazole. Some experts recommend concomitant therapy with steroids to decrease the inflammatory response from dying parasites.

312. A 4-year-old child is observed to hold his eyelids open with his fingers and to close one eye periodically, especially in the evening. He has some trouble swallowing his food. He usually appears sad, although he laughs often enough. He can throw a ball, and he runs well. Which of the following is most likely to aid in the diagnosis?

c. Effect of a test dose of edrophonium Myasthenia gravis is an autoimmune disorder in which circulating acetylcholine receptor-binding antibodies result in neuromuscular blockade. The earliest signs of myasthenia gravis are ptosis and weakness of the extraocular muscles, followed by dysphagia and facial muscle weakness. The distinguishing hallmark of this disease is rapid fatiguing of the involved muscles. Cholinesterase inhibitors are the primary therapeutic agents. Other therapeutic modalities for myasthenia gravis include immunosuppression, plasmapheresis, thymectomy (an enlarged thymus is frequently seen on chest x-ray), and treatment of hypothyroidism. CPK should be normal.

435. A 4-year-old child has mental retardation, shortness of stature, brachydactyly (especially of the fourth and fifth digits), and obesity with round facies and short neck. The child is followed by an ophthalmologist for subcapsular cataracts, and has previously been noted to have cutaneous and subcutaneous calcifications, as well as perivascular calcifications of the basal ganglia. What is the diagnosis?

c. Elevated concentrations of parathyroid hormone The patient with the features listed likely has *pseudohypoparathyroidism* (Albright hereditary osteodystrophy). Such patients have chemical findings of hypoparathyroidism (low calcium, high phosphorus), but parathyroid hormone levels are high, indicating resistance to the action of this hormone. Parathyroid hormone infusion does not produce a phosphaturic response. Phenotypically, these patients demonstrate shortness of stature with delayed bone age, mental retardation, increased bone density throughout the body (especially evident in the skull), brachydactyly (especially of the fourth and fifth digits), obesity with round facies and short neck, subcapsular cataracts, cutaneous and subcutaneous calcifications, and perivascular calcifications of the basal ganglia.

426. The parents of a 1-month-old infant bring him to the emergency center in your local hospital for emesis and listlessness. Both of his parents wanted a natural birth, so he was born at home and has not yet been to see a physician. On examination, you find a dehydrated, listless, and irritable infant. Although you don't have a birth weight, the parents do not feel that he has gained much weight. He has significant jaundice. His abdominal examination is significant for both hepatomegaly and splenomegaly. Laboratory values include a total bilirubin of 15.8 mg/dL and a direct bilirubin of 5.5 mg/dL. His liver function tests are elevated and his serum glucose is 38 mg/dL. Serum ammonia is normal. A urinalysis is negative for glucose, but it has a "mouse-like" odor. These findings are consistent with which of the following conditions?

c. Galactosemia The patient has classic findings of galactosemia. Galactose is a component of lactose, found in breast milk and most infant formulas. Symptoms of galactosemia occur in the first weeks of life. While screening for classic galactosemia typically is part of the newborn metabolic panel, patients fitting the clinical presentation as outlined in the question must be evaluated promptly. Signs and symptoms in addition to those presented in the vignette include cataracts and ascites. While three different errors in galactose metabolism are known, most cases result from the deficiency in galactose-1-phosphate uridyl transferase.

20. An 8-hour-old infant develops increased respiratory distress, hypothermia, and hypotension. A complete blood count (CBC) demonstrates a white blood cell (WBC) count of 2500/μL with 80% bands. The chest radiograph is shown below. Which of the following is the most likely diagnosis?

c. Group B streptococcal pneumonia

393. Two weeks after a viral syndrome, a 2-year-old child develops bruising and generalized petechiae, more prominent over the legs. No hepatosplenomegaly or lymph node enlargement is noted. The examination is otherwise unremarkable. Laboratory testing shows the patient to have a normal hemoglobin, hematocrit, and white blood cell (WBC) count and differential. The platelet count is 15,000/μL. Which of the following is the most likely diagnosis?

c. Idiopathic (immune) thrombocytopenic purpura (ITP) In children, ITP is the most common form of thrombocytopenic purpura. In most cases, a preceding viral infection can be noted. No diagnostic test identifies this disease; exclusion of the other diseases listed in the question is necessary. ITP and TTP are both blood disorders that involve platelet counts. In ITP, there is a failure of the blood to clot, while TTP results from the formation of too many blood clots which lead to overused platelets. Though each condition has a specific mechanism, both have the same end symptoms, which are easy bruising and bleeding. Treatment of TTP involves blood replacement therapy to correct the imbalance of the blood components, while ITP can be treated with oral steroids. The cause for ITP cannot be determined, while TTP is usually caused by spontaneous platelet aggregation.

175. A 14-year-old girl, angry at her mother for taking away her MP3 player, takes an unknown quantity of a friend's pills. Within the first hour she is sleepy, but in the emergency center she develops a widened QRS complex on her electrocardiogram (ECG), hypotension, and right bundle branch block. The therapy you would initiate for this ingestion is which of the following?

c. Intensive care unit (ICU) admission, close monitoring, and possible Fab antibody fragments

50. A 15-month-old toddler continues to wake up crying every night. Her parents give her a nighttime bottle, rock her, and sing to her to help her go back to sleep. Her parents are exhausted and ask you if she is having bad dreams.

c. Learned behavior

307. A 6-year-old child has had repeated episodes of otitis media. She undergoes an uneventful surgical placement of pressure-equalization (PE) tubes. In the recovery room she develops a fever of 40°C (104°F), rigidity of her muscles, and metabolic and respiratory acidosis. Which of the following is the most likely explanation for her condition? What to treat with?

c. Malignant hyperthermia Treat with muscle relaxant dantrolene. Dantrolene achieves this by inhibiting Ca2+ ions release from sarcoplasmic reticulum stores by antagonizing ryanodine receptors.

Name the heart condition A new patient to your practice, a happy 10-year-old girl has a history of hyperextensible skin and easy bruising. She often has sprains and joint dislocations when she exercises.

c. Mitral valve prolapse Mitral valve prolapse is seen in patients with Ehlers-Danlos syndrome. Patients with Ehlers-Danlos syndrome have hyperextensibility and easy bruising, joint hypermobility (leading to joint dislocations and sprains), skin that is velvety to touch, and tissue fragility. Six different kinds are described with "classic" and "hypermobility" forms predominating.

351. A 10-year-old boy was healthy until about 10 days ago when he developed 7 days of fever, chills, severe muscle pain, pharyngitis, headache, scleral injection, photophobia, and cervical adenopathy. After 7 days of symptoms he seemed to get better, but yesterday he developed fever, nausea, emesis, headache and mild nuchal rigidity. Cerebrospinal fluid (CSF) shows 200 white blood cells (WBC) per microliter (all monocytes) and an elevated protein. What is the diagnosis?

c. Most cases are mild or subclinical Leptospirosis is the most common zoonotic infection worldwide, and is often a mild or subacute illness, frequently escaping detection. Usually a history of exposure to dogs, cats, livestock, rats, or other wild animals is obtained.

Questions 103 to 107 103. A 1-year-old boy has been admitted three times in the past with abscess formation requiring incision and drainage. He is now admitted for surgical drainage of a hepatic abscess identified on ultrasound most appropriate diagnostic laboratory test

c. Nitroblue tetrazolium (NBT) test CGD

31. An 8-month-old infant arrives to the emergency department (ED) with a 2-day history of diarrhea and poor fluid intake. Your quick examination reveals a lethargic child; his heart rate is 180 beats per minute, his respiratory rate is 30 breaths per minute, and his blood pressure is low for age. He has poor skin turgor, 5-second capillary refill, and cool extremities. Which of the following fluids is most appropriate management for his condition?

c. Normal saline

110. A 2-hour-old full-term newborn infant is noted by the nursing staff to be having episodes of cyanosis and apnea. Per nursery protocol they place an oxygen saturation monitor on him. When they attempted to feed him, his oxygen levels drop into the 60s. When he is stimulated and cries, his oxygen levels increase into the 90s. Which of the following is the most important next step to quickly establish the diagnosis?

c. Passage of catheter into nose It is important to make the diagnosis of choanal atresia quickly because it responds to treatment but can be lethal if unrecognized and untreated. Most neonates are obligate nose breathers and cannot breathe adequately through their mouths. The diagnosis can be made by failure to pass a catheter through the nose to the pharynx or by checking for fog developing on a cold metal instrument placed under each naris.

198. A 15-year-old adolescent female comes to be evaluated for syncopal episodes. Her only other medical problem is congenital deafness. She notes through a signing interpreter that syncopal episodes happen during stressful or emotional situations and that they started only within the past year or so. What is her heart condition?

c. Prolonged QT syndrome Prolonged QT syndrome occurs in 1:10,000 to 1:15,000 children, usually first causing syncopal episodes in late childhood or adolescence. During the syncope episode, arrhythmias may be noted, including ventricular fibrillation. These episodes may result in death. QT intervals are elongated on ECG. An autosomal recessive form associated with deafness (Jervell-Lange-Nielsen syndrome) and an autosomal dominant form (Romano-Ward syndrome) have been described.

7. A patient comes to your office for a hospital follow-up. You had sent him to the hospital 3 weeks earlier for persistent fevers but no other symptoms; he was diagnosed with endocarditis and is currently being treated appropriately. Advice to this family should now include which of the following?

c. Provide the child with antibiotic prophylaxis for dental procedures.

433. A 12-year-old girl has a solitary thyroid nodule found on routine examination; she has no symptoms. Which of the following is the most appropriate next step for this patient?

c. Serum thyroid function tests The first step in the management of a pediatric patient with a solitary thyroid nodule is measurement of thyroid function. A variety of diagnostic procedures are available to evaluate the patient with a nodule, including measurement of thyroid function, anti-thyroid antibody determination, ultrasound, and radionuclide uptake and scan. Depending on the results of these tests, a fine needle aspiration may ultimately be required

17. You are called to the ER to see one of your patients. The father of this 14-year-old mildly retarded child says that he found the child about 20 minutes ago in the neighbor's garden shed with an unknown substance in his mouth. The child first had a headache, but then became agitated and confused; while you are talking to the father in the ER the child begins to have a seizure and dysrhythmia on the cardiac monitor. The blood gas demonstrates a severe metabolic acidosis. Which of the following agents is most likely the culprit?

c. Sodium cyanide

173. A 6-month-old comatose infant has multiple broken bones in various stages of healing, a bulging anterior fontanelle, and retinal hemorrhages.

c. Subdural hemorrhage Subdural hematomas are commonly seen as part of abusive head trauma, formerly known as shaken baby syndrome. This lesion occurs when the bridging cortical veins that drain the cerebral cortex have been ruptured, leading to a collection of blood between the dura and the cerebral mantle. Repeated trauma can lead to additional collections of blood. In many children, additional findings of abuse such as broken bones, bruises, and retinal hemorrhages are found.

315. A 5-month-old child was normal at birth, but the family reports that the child does not seem to look at them any longer. They also report the child seems to "startle" more easily than he had before. Testing of his white blood cells (WBCs) identifies the absence of β-hexosaminidase A activity, confirming the diagnosis of which of the following?

c. Tay-Sachs disease

284. The mother of a 2-year-old male child states that she has noticed white, cheeselike material arising from his foreskin and also that he cannot fully retract the foreskin behind the glans penis. Which of the following is the correct advice for this parent?

c. The child is normal. This is smegma

42. A 5-month-old child with poor growth presents to the ER with generalized tonic-clonic seizure activity of about 30-minute duration that stops upon the administration of lorazepam. Which of the following historical bits of information gathered from the mother is most likely to lead to the correct diagnosis in this patient?

c. The mother has been diluting the infant's formula to make it last longer

366. A bat is found in the bedroom of a 4-year-old patient while the boy is sleeping. The family and the patient deny close contact with or bites from the bat. Which of the following is a correct statement regarding this situation?

c. The patient should be started on the rabies vaccine series. Bats have been associated with the majority of the rabies cases in the United States, with most victims not having an obvious bite. Therefore, the CDC recommends that post exposure prophylaxis (PEP) be considered for any close encounter with a bat, even without an obvious bite, such as the one described above. PEP consists of wound cleansing (if there is a wound), rabies immuno globulin (RIG), and the five injection rabies vaccine series. Modifications may be made for patients who have received PEP in the past. Rabies is almost uniformly fatal; many different medications have been attempted, including ribavirin and a-interferon, without success. Of the six reported cases of survival from rabies infection, five had some of the recommended PEP.

190. A cyanotic newborn is suspected of having congenital heart disease. He has an increased left ventricular impulse and a holosystolic murmur along the left sternal border. The ECG shows left-axis deviation and left ventricular hypertrophy (LVH). Which of the following is the most likely diagnosis?

c. Tricuspid atresia Patients with tricuspid atresia typically have a hypoplastic right ventricle, and therefore the ECG shows left-axis deviation and LVH; this translates to a left ventricular impulse on physical examination. Almost all other forms of cyanotic congenital heart disease are associated with elevated pressures in the right ventricle and increased right ventricular impulse.

29. A 2-year-old boy has been vomiting intermittently for 3 weeks and has been irritable, listless, and anorectic. His use of language has regressed to speaking single words. In your evaluation of this patient, which of the following is the most reasonable diagnosis to consider?

c. Tuberculous meningitis

276. A 12-year-old who has several weeks of abdominal pain and black stools

c. Upper GI endoscopy The second patient's story is concerning for PUD. An upper GI series may show findings suggestive of the diagnosis, but endoscopy is the preferred diagnostic method, as it allows biopsy for microscopy and culture, and direct visualization.

Questions 335 to 337 335. A 15-year-old girl has an acute, recurrent, pulsatile headache localized behind the eyes that tends to occur more frequently around menses. She has no symptoms that occur prior to the headache; her neurologic examination is normal.

c. Vascular headache (migraine) While the typical scotomata discussed in adult migraine is not normally associated in children with migraine, pediatric migraines may have a nonspecific prodrome consisting of a change in mood, temperament, or appetite.

98. Measles in developing countries What supplement is indicated for baby?

c. Vitamin A

272. A 17-month-old toddler has been irritable over the past month. She now refuses to walk and seems to have tenderness in both of her legs. She has had a low-grade fever, and she has petechiae on her skin and mucous membranes. She has a small cut that has not healed well. Radiographs of the legs reveal generalized bony atrophy with epiphyseal separation.

c. Vitamin C

What is the diagnosis? 168. A child's left arm and leg seem bigger than those on the right. In addition, the child has aniridia. None of the family members have aniridia or hemihypertrophy, nor do they know of anyone else in the family with these conditions.

c. Wilms tumor (WAGR) *Aniridia* is found in 1% to 2% of children with Wilms tumor. *Genitourinary anomalies* are found in 4% to 5%, and *hemihypertrophy* is associated with this tumor in 2% to 3% of patients. Wilms tumor is the most common primary renal malignancy in childhood. Prognosis is generally good.

188. A 4-year-old girl is brought to the pediatrician's office. Her father reports that she suddenly became pale and stopped running while he had been playfully chasing her and her pet Chihuahua. After 30 minutes, she was no longer pale and wanted to resume the game. She has never had a previous episode and has never been cyanotic. Her physical examination was normal, as were her chest x-ray and echocardiogram. An ECG showed the pattern seen on the next page, which indicates which of the following?

c. Wolff-Parkinson-White syndrome The child described in the question has no cyanosis or murmur, no cardiac or pulmonary vascular abnormalities by chest x-ray, and no evidence of structural anomalies by echocardiogram, and therefore is unlikely to have an underlying gross anatomic defect. These patients have an aberrant atrioventricular conduction pathway, which causes the early ventricular depolarization appearing on the ECG as a shortened PR interval. The initial slow ventricular depolarization wave is referred to as the delta wave. Seventy percent of patients with WPW syndrome have single or repeated episodes of paroxysmal supraventricular tachycardia, which can cause the symptoms described in the question.

3. A child is brought to your clinic for a routine examination. She can put on a T-shirt but requires a bit of help dressing otherwise. She can copy a circle well but has difficulty in copying a square. Her speech is understandable and she knows four colors. She balances proudly on each foot for 2 seconds but is unable to hold the stance for 5 seconds. Which of the following is the most likely age of this child?

d. 4 years

486. A 16-year-old girl is in your office for a preparticipation sports examination. She plans to play soccer in the fall, and needs her form filled out. Which of the following history or physical examination findings is usually considered a contraindication to playing contact sports?

d. Absence of a single eye Having one eye is typically seen as a contraindication to playing contact sports like football or soccer. Other reasons to recommend against contact sports participation include hemophilia, single kidney, and unexplained syncope (until evaluated).

304. A 6-year-old girl with a complaint of "dark urine"; she has a blood pressure of 120/80 mm Hg.

d. Acute glomerulonephritis With acute glomerulonephritis, oliguria (often presenting with dark, cola-colored urine) frequently occurs as a direct consequence of the disease process itself; on occasion, it can be profound, with virtual anuria for several days. During this period of time, it is vital to monitor and restrict fluid intake lest massive edema, hypervolemia, and even pulmonary edema and death occur.

138. You are advised by the obstetrician that the mother of a baby she has delivered is a carrier of hepatitis B surface antigen (HBsAg-positive). Which of the following is the most appropriate action in managing this infant?

d. Administer hepatitis B immune globulin and hepatitis B vaccine to the infant. It is not necessary to isolate infants born to carriers of HBsAg, and screening of neonates for HBsAg is not indicated. Testing for HBsAg and anti-HBsAg at least 1 month after the third dose of hepatitis B vaccine will determine the efficacy of these measures.

Questions 79 to 83 Excess vitamin intake has been shown to have deleterious effects. Match the vitamin with the toxic effect. Each lettered option may be used once, more than once, or not at all.. 79. Vitamin A

d. An irritable 8-year-old child with headache, vomiting, alopecia, dry/itchy skin with desquamation of the palms and soles, hepatosplenomegaly, and swelling of the bones.

38. As a city public health officer, you have been charged with the task of screening high-risk children for lead poisoning. Which of the following is the best screen for this purpose?

d. Blood lead level

163. A preterm infant is now 7 weeks old. She was intubated for 2 weeks and was weaned off oxygen at 3 weeks of age. You are about to leave your office for Thanksgiving holiday when the emergency room calls to tell you she has new hypoxia, respiratory distress, wheezes, and runny nose. A chest radiograph reveals patchy infiltrates and hyperexpansion in both lung fields. The newborn's 2-year-old sibling has an upper respiratory infection.

d. Bronchiolitis Bronchiolitis is a very common viral infection most often caused by respiratory syncytial virus. It is most often seen in the winter months with symptoms of wheezing, hypoxia, and respiratory distress seen in younger children; often an older sibling has milder, upper respiratory symptoms. Premature infants, infants with congenital heart disease, infants with a variety of lung disorders, and infants with immune system defects are at higher risk of severe complications. Diagnosis is made by clinical history and/or detection of the viral antigen in nasal secretions; treatment is supportive.

320. A 2-year-old boy has been doing well despite his diagnosis of tetralogy of Fallot. He presented to an outside ER a few days ago with a complaint of an acute febrile illness for which he was started on a "pink antibiotic." His mother reports that for the past 12 hours or so he has had a headache and is more lethargic than normal. On your examination he seems to have a severe headache, nystagmus, and ataxia. Which of the following would be the most appropriate first test to order?

d. CT or MRI of the brain The patient in the question has a brain abscess, a condition more commonly seen in patients with cardiac defects that have right-to-left shunts associated with them. The diagnostic tool of choice is imaging, and either CT or MRI is indicated. Lumbar puncture would be contraindicated in this patient until after imaging (the patient is at risk for brain herniation) and the CSF (and blood) cultures are usually negative.

316. The family of a 4-year-old boy has just moved into your area. The child was recently brought to the emergency department (ED) for an evaluation of abdominal pain. Although appendicitis was ruled out in the ED and the child's abdominal pain has resolved, the ED physician requested that the family follow up in your office to evaluate an incidental finding of an elevated creatine kinase. The family notes that he was a late walker (began walking independently at about 18 months of age), that he is more clumsy than their daughter was at the same age (especially when trying to hold onto small objects), and that he seems to be somewhat sluggish when he runs, climbs stairs, rises from the ground after he sits, and rides his tricycle. A thorough history and physical examination are likely to reveal which of the following?

d. Cataracts The child in the question appears to have myotonic muscular dystrophy. An elevated creatinine kinase (especially in the preclinical phase) often is found, and psychomotor retardation can be the presenting complaint (but may be identified only in retrospect). Ptosis, baldness, hypogonadism, facial immobility with *distal* muscle wasting (in older children), and neonatal respiratory distress (in the newborn period) are major features of this disorder. Cataracts are commonly seen, presenting either congenitally or at any point during childhood. The prominence of distal muscle weakness in this disease is in contrast to the proximal muscle weakness seen in most other forms of myopathies. The diagnosis is confirmed by a molecular blood test. Seizures are not a feature of myotonic dystrophy. Enlarged gonads are associated with fragile X syndrome, and hirsutism is found (among other things) in children with congenital adrenal hyperplasia.

410. An otherwise healthy 17-year-old complains of swollen glands in his neck and groin for the past 6 months and an increasing cough over the previous 2 weeks. He also reports some fevers, especially at night, and possibly some weight loss. On examination, you notice that he has nontender cervical, supraclavicular, axillary, and inguinal nodes, no hepatosplenomegaly, and otherwise looks to be fairly healthy. Which of the following would be the appropriate next step?

d. Chest radiograph The patient in this question, especially with the pulmonary findings, may have Hodgkin disease. Systemic symptoms of Hodgkin disease include fever, night sweats, malaise, weight loss, and pruritus. Although a biopsy of the node may prove to be necessary at some point, the first step would involve a radiograph, which may show mediastinal mass suspicious for Hodgkin disease.

201. You are asked to evaluate an infant born vaginally 3 hours previously to a mother whose only pregnancy complication was poorly controlled gestational diabetes. The nursing staff noticed that the infant was breathing abnormally. On examination, you find that the infant is cyanotic, has irregular, labored breathing, and has decreased breath sounds on the right side. You also note decreased tone in the right arm. You provide oxygen and order a stat portable chest radiograph, which is normal. Which of the following studies is most likely to confirm your diagnosis?

d. Chest ultrasound Infants born to mothers with gestational diabetes are at risk for being large for their gestational age and thus at increased risk for peripheral nerve injuries such as Erb-Duchenne and phrenic nerve paralysis. An ultrasound or fluoroscopy of the chest would reveal asymmetric diaphragmatic motion in a seesaw manner.

225. A 6-week-old child arrives with a complaint of "breathing fast" and a cough. On examination you note the child to have no temperature elevation, a respiratory rate of 65 breaths per minute, and her oxygen saturation to be 94%. Physical examination also is significant for rales and rhonchi. The past medical history for the child is positive for an eye discharge at 3 weeks of age, which was treated with a topical antibiotic drug. Which of the following organisms is the most likely cause of this child's condition?

d. Chlamydia trachomatis The sites of infection in infants are the conjunctivae and the lungs, where chlamydiae cause inclusion conjunctivitis and afebrile pneumonia, respectively, in infants between 2 and 12 weeks of age.

13. A 6-year-old boy is often teased at school because he has stooled in his underwear almost daily for the last 3 months. He was toilet trained at 2 years of age without difficulty, but over the last 2 years he had developed ongoing constipation. His family is frustrated because they cannot believe him when he says "I didn't know I had to go." He is otherwise normal; school is going well, and his home life is stable. His only finding on examination is significant for stool in the rectal vault. The plain radiograph of his abdomen is shown. Initial management of this problem should include which of the following?

d. Clear fecal impaction and short-term stool softener use

56. A 9-year-old girl presents during summer break with an area of erythematous, firm, and slightly swollen skin at the corner of her mouth and extending to her cheek. The area is not tender.

d. Cold panniculitis

159. A newborn has microcephaly, intracranial calcifications, hepatosplenomegaly, and marked hyperbilirubinemia and thrombocytopenia

d. Congenital cytomegalovirus

200. An infant previously diagnosed with a large muscular VSD comes to the office with complaints from the mother of fatigue and poor feeding over the past month. You note the child has not gained weight since the previous visit 2 months ago. The child is apathetic, tachypneic, and has wheezes and crackles on lung auscultation. What is her heart condition?

d. Congestive heart failure Congestive heart failure can result from a number of causes, including congenital heart disease, Kawasaki disease, metabolic cardiomyopathies, arrhythmias, and viral myocarditis. The case presented is typical of an infant with heart failure. Such infants are weak, diaphoretic, have poor weight gain, and may be tachypneic with retractions. Lung findings can include crackles, wheezes, or both. Prompt evaluation by a pediatric cardiologist is imperative.

427. A 2-month-old boy is admitted to the hospital for failure to thrive. You note him to have fatty stools, and consider cystic fibrosis in your differential diagnosis. You order pilocarpine iontophoresis, but the laboratory calls to say they could not collect enough sweat from the infant. Which of the following is another way to make the diagnosis of cystic fibrosis?

d. DNA testing for cystic fibrosis transmembrane regulator (CFTR) mutations

470. A newborn with hypoglycemia, hypocalcemia, and hypoplastic lower extremities.

d. Diabetic embryopathy Infants born to diabetic mothers are frequently macrosomic and may become hypoglycemic. However, they can have many other problems as well, including cardiac septal hypertrophy, congenital heart disease, caudal regression, vertebral defects, and a single umbilical artery.

Double bubble sign + polyhydramnios =

d. Duodenal atresia

187. A 2-year-old child with minimal cyanosis has an S3 and S4 (a quadruple rhythm), a systolic murmur in the pulmonic area, and a middiastolic murmur along the lower left sternal border. An ECG shows right atrial hypertrophy and a ventricular block pattern in the right chest leads. Which of the following is the most likely diagnosis?

d. Ebstein anomaly A quadruple rhythm associated with the murmur of tricuspid regurgitation and a middiastolic murmur at the lower left sternum suggests the diagnosis of Ebstein anomaly (downward displacement of the tricuspid valve).

352. A previously healthy 8-year-old boy has a 3-week history of low-grade fever of unknown source, fatigue, weight loss, myalgia, and headaches. On repeated examinations during this time, he is found to have developed a heart murmur, petechiae, and mild splenomegaly. Which of the following is the most likely diagnosis?

d. Endocarditis The presentation of infective endocarditis can be quite variable, ranging from prolonged fever with few other symptoms to an acute and severe course with early toxicity. A high index of suspicion is necessary to make the diagnosis quickly. Identification of the causative organism (frequently Streptococcus sp or Staphy-lococcus sp) through multiple blood cultures is imperative for appropriate treatment. Echocardiography may identify valvular vegetations and can be predictive of impending embolic events, but a negative echocardiogram does not rule out endocarditis.

354. The 3-year-old sister of a newborn baby develops a cough diagnosed as pertussis by nasopharyngeal culture. The mother gives a history of having been immunized as a child. Which of the following is a correct statement regarding this clinical situation?

d. Erythromycin should be administered to the infant. Newborn infants exposed to pertussis are at considerable risk of being infected regardless of the immune status of the mother. In contrast to other childhood infectious diseases, pertussis is not entirely prevented by transplacentally acquired antibody. Erythromycin achieves high concentrations in respiratory secretions and is effective in eliminating organisms from the respiratory tract of patients.

78. A 17-year-old is brought into the ED by his friends at about 10:00 AM. They were at a party the night before and drank some "homemade" alcohol. He is disoriented and confused, and has an anion-gap acidosis. He begins to have seizures

d. Ethanol Treatment includes emptying the stomach by inducing emesis or by gastric lavage, the IV infusion of ethanol to saturate the enzyme systems that convert methanol to toxins, and, in severe poisoning, the use of hemodialysis to remove the methanol.

41. A mentally retarded 14-year-old boy has a long face, large ears, micropenis, and large testes. Chromosome analysis is likely to demonstrate which of the following?

d. Fragile X syndrome

130. The newborn pictured below was born at home and has puffy, tense eyelids; red conjunctivae; a copious amount of purulent ocular discharge; and chemosis 2 days after birth. Which of the following is the most likely diagnosis?

d. Gonococcal ophthalmia Erythromycin drops are helpful in preventing gonococcal infections. NOT chlamydia. Remember as well that gonococcal eye infections occur 2-5 days after birth. Chlamydial infections occur 5-14 days after birth.

283. A 6-month-old infant has poor weight gain, vomiting, episodic fevers, and chronic constipation. Laboratory studies reveal a urinalysis with a pH of 8.0, specific gravity of 1.010, 1+ glucose, and 1+ protein. Urine anion gap is normal. Serum chemistries show a normal glucose and a normal albumin with a hyperchloremic metabolic acidosis. Serum phosphorus and calcium are low. What is the best diagnosis to explain these findings?

d. Hereditary Fanconi syndrome The nonspecific findings of anorexia, polydipsia and polyuria, vomiting, and unexplained fevers, along with the more specific laboratory abnormalities of glucosuria but normal blood sugar, abnormally high urine pH in the face of mild or moderate serum hyperchloremic metabolic acidosis, and mild albuminuria in the presence of normal serum protein and albumin, suggest Fanconi syndrome.

179. For the past year, a 12-year-old boy has had recurrent episodes of swelling of his hands and feet, which has been getting worse recently. These episodes occur following exercise and emotional stress, last for 2 to 3 days, and resolve spontaneously. The last episode was accompanied by abdominal pain, vomiting, and diarrhea. The results of routine laboratory workup are normal. An older sister and a maternal uncle have had similar episodes, but they were not given a diagnosis. He presents today with another episode as shown in the photographs on the next page. Which of the following is the most likely diagnosis?

d. Hereditary angioedema Although hereditary angioedema is relatively rare as a cause of edema, the recurrent episodes in late childhood, the normal laboratory results, and the family history in the vignette make the other choices less likely. Hereditary angioedema, transmitted as an autosomal dominant trait, is a result of inadequate function (owing to either deficient quantity or quality) of C1 esterase inhibitor (C1-INH), involved in the first step in the complement cascade, which results in the excessive production of a vasoactive kinin. In addition to otherwise asymptomatic subcutaneous edema, edema can occur in the gastrointestinal tract and produce the symptoms mentioned in the question. Laryngeal edema with airway obstruction can also occur; a change in the tone of the voice and difficulty swallowing secretions are symptoms of impending airway compromise and require emergent medical care.

293. A 6-year-old girl is brought to the emergency room because her urine is red. She has been healthy her whole life, and has recently returned from an outing with her grandmother to a local amusement park. Her urine dip for heme is positive, suggesting which of the following is a possibility?

d. Presence of myoglobin

336. A 7-year-old boy has chronic, worsening headache without preceding symptoms. He complains of emesis in the morning before breakfast for the last 2 weeks.

d. Increased intracranial pressure Worsening headaches with nausea and emesis (particularly early morning emesis) are concerning for increased intracranial pressure from a mass lesion. Other associated findings may be decreased school performance, behavioral changes, or focal neurologic deficits. Papilledema may be present. Imaging would be necessary with this presentation.

215. You are awakened in the night by your 2-year-old son, who has developed noisy breathing on inspiration, marked retractions of the chest wall, flaring of the nostrils, and a barking cough. He has had a mild upper respiratory infection (URI) for 2 days. Which of the following therapies is indicated?

d. Inhaled epinephrine and a dose of steroids Agents causing croup include parainfluenza types 1 and 3, influenza A and B, RSV, and occasionally other viruses. Treatment is usually supportive, but racemic epinephrine and corticosteroids reduce the length of time in the emergency room and hospitalizations.

39. A 15-year-old is participating in high school football practice in August in Texas. He had complained of headache and nausea earlier in practice, but kept playing after a cup of water. He is now confused and combative. He is dizzy and sweating profusely. His temperature is 41°C (105.8°F). Therapy should consist of which of the following?

d. Initiate whole body cold water immersion

290. A 6-week-old child is being evaluated for a fever of unknown etiology. As part of the laboratory evaluation, a urine specimen was obtained that grew E coli with a colony count of 2000/μL. These findings would be definite evidence of a urinary tract infection if which of the following is true about the sampled urine?

d. It is from a suprapubic tap

26. A 5-year-old boy presents with the severe rash shown in the photographs. The rash is pruritic, and it is especially intense in the flexural areas. The mother reports that the symptoms began in infancy (when it also involved the face) and that her 6-month-old child has similar symptoms. Which of the following is the most appropriate treatment of this condition?

d. Moisturizers and topical steroids

73. You are called to the delivery room. A newborn infant seems lethargic and has poor tone with only marginal respiratory effort, but his heart rate is above 100 beats per minute. The mother had an uncomplicated pregnancy, and delivery was uncomplicated and vaginal 10 minutes after spontaneous rupture of membranes. The mother received only pain medications while in labor.

d. Naloxone (Narcan)

109. A newborn is noted to be quite jaundiced at 3 days of life. Laboratory data demonstrate his total bilirubin to be 17.8 mg/dL (direct bilirubin is 0.3 mg/dL). Which of the following factors is associated with an increased risk of neurologic damage in a jaundiced newborn?

d. Neonatal sepsis Metabolic acidosis also reduces binding of bilirubin, and neonatal sepsis interrupts the blood-brain barrier, thus allowing diffusion of bilirubin into the brain. Administration of phenobarbital has been used to induce glucuronyl transferase in newborn infants and can reduce, rather than exacerbate, neonatal jaundice.

451. An otherwise healthy 18-year-old girl with medullary thyroid carcinoma What are the PO4 and Ca levels in this patient?

d. Normal PO4, normal Ca Medullary carcinoma of thyroid arises from the C cells of the thyroid. These tumors secrete excessive amounts of calcitonin, and, accordingly, the concentration of this hormone in the blood is increased.

205. A 2-year-old girl is playing in the garage with her Chihuahua, only partially supervised by her father, who is weed-whacking around the garden gnomes in the front yard. He finds her in the garage, gagging and vomiting. She smells of gasoline. In a few minutes she stops vomiting, but later that day she develops cough, tachypnea, and subcostal retractions. She is brought to your emergency center. Which of the following is the most appropriate first step in management?

d. Perform pulse oximetry and arterial blood gas Treatment is symptomatic, sometimes requiring intubation and mechanical ventilation. Induction of emesis is contraindicated, as this may cause further aspiration.

348. A 2-year-old child is admitted to your hospital team. The child's primary care doctor has been following the child for several days and has noted her to have had high fever, peeling skin, abdominal pain, and a bright red throat. You are concerned because two common pediatric problems that could explain this child's condition have overlapping presenting signs and symptoms. Which of the following statements comparing these two diseases in your differential is true?

d. Pharyngeal culture aids in the diagnosis of one of the conditions. The most serious complication of Kawasaki disease and scarlet fever is cardiac involvement. Erythrogenic toxin-producing group A β-hemolytic streptococci is the agent responsible for scarlet fever. Isolation of the organism from the nasopharynx and a rise in antistreptolysin titers will confirm the diagnosis.

263. A 16-year-old male, despondent over a recent breakup, tries to commit suicide by taking an unknown quantity of an unknown material he found at home. He is brought to the emergency center by his parents within 30 minutes of the ingestion. For which of the following household materials and medications should he be given activated charcoal as part of his emergency center treatment?

d. Phenobarbital The absorption of certain toxins from the GI tract is diminished by the use of activated charcoal administered during the first few hours after the ingestion. The typical dose is 1 g/kg, or 10 to 30 g for a child or 30 to 100 g for an adult. Activated charcoal exerts its effect by adsorbing particles of toxin on its surface.

423. During a routine well-child examination a 10-year-old girl reports that she has occasional headache, "racing heart," abdominal pain, and dizziness. Her mother states that she has witnessed one of the episodes, which occurred during an outing at the mall, and reported the child to be pale and to have sweating as well. Other than some hypertension, she has a normal physical examination. Evaluation of this child is most likely to result in which of the following diagnoses?

d. Pheochromocytoma The child in the question has all of the classic symptoms of childhood pheochromocytoma. In adults, the episodes of hypertension are more paroxysmal than in children, where the hypertension is more sustained. While it is an unusual diagnosis in children, pheochromocytoma must be considered in the evaluation of a patient with hypertension who has intermittent symptoms described. Pheochromocytoma can be associated with tuberous sclerosis, Sturge-Weber syndrome, ataxia-telangiectasia, and it can be inherited as an autosomal dominant trait

429. The state laboratory calls your office telling you that a newborn infant, now 8 days old, has an elevated thyroid stimulating hormone (TSH) and low thyroxin (T4) on his newborn screen. If this condition is left untreated, the infant is likely to demonstrate which of the following in the first few months of life?

d. Prolonged jaundice Signs of congenital hypothyroidism include constipation, prolonged jaundice, sluggishness, poor feeding, apnea, choking, macroglossia, and excessive sleepiness. The physical examination is usually normal early on except for mild jaundice and a distended abdomen in a sleepy infant.

Name the heart condition A 15-year-old boy comes to your office for a Special Olympics sports physical. His height is in the 3rd to 5th percentile and his weight is in the 50th percentile. Physical examination reveals a young man with shieldlike chest, cryptorchidism, low-set and malformed ears, ptosis, and pectus excavatum.

d. Pulmonic stenosis Noonan syndrome, the "male Turner syndrome," occurs in both sexes. The most common features include short stature, downslanting palpebral fissures, ptosis, low set and malformed ears, webbed neck, shieldlike chest, pulmonic stenosis, and cryptorchidism. Mental retardation is seen in one-fourth of affected individuals. It is associated with advanced paternal age.

45. A nurse calls you to evaluate an African American newborn whom she thinks has a bacterial skin infection. The areas in question have many scattered pustules full of a milky fluid. Upon examining pustules, they easily wipe away, revealing a small hyperpigmented macule.

d. Pustular melanosis

221. You are asked by a colleague to evaluate a 5-year-old boy as a second opinion. He has a history of chronic and recurrent upper respiratory tract infections, several admissions to the hospital for pneumonia, and three surgeries for PE tubes for chronic otitis media. Of note is a right-sided heart on repeated radiographs. Convinced you know the diagnosis based on history alone, you confirm your diagnosis with a biopsy of the nasal mucosa. You expect to find which of the following?

d. Random orientation of cilia Patients with primary ciliary dyskinesia (PCD) (immotile cilia syndrome) have dysfunctional cilia and, as such, have abnormal airway clearance. Described structural aberrations include abnormal cytoskeletal proteins and defects in the dynein arms. Kartagener syndrome (the triad of situs inversus, chronic sinusitis and otitis media, and airway disease) is associated with primary ciliary dyskinesia; approximately 50% of patients with the latter also have the former.

What problems can occur as a result of cleft lip and cleft palate?

d. Recurrent otitis media and hearing loss are likely complications. The infant pictured has bilateral cleft lip and palate. Evaluation for other structural and chromosomal abnormalities is indicated. Although affected infants are likely to have feeding problems initially, these problems usually can be overcome by feeding in a propped-up position and using special nipples. Complications include recurrent otitis media and hearing loss as well as speech defects, which may be present despite good anatomic closure.

69. A 2-year-old refuses to walk, has fever, has significant pain with external rotation of the right leg, and has an elevated WBC count.

d. Septic arthritis of the hip

85. A woman without prenatal care has a diet low in green vegetables and enriched grain products.

d. Spina bifida Neural tube defects

6. A mother brings to your office an article from the Internet suggesting that infants in day care have a statistically higher incidence of upper respiratory infections (p < 0.05) as compared to children not in day care. You explain to her that this means which of the following?

d. The odds are less than 1 in 20 that the differences in upper respiratory infection rates observed were only a chance variation.

493. A 19-year-old male has recurrent episodes of painful, erythematous, small vesicles, and ulcers on his glans penis.

d. Tzanck preparation for multinucleated giant cells

489. A 19-year-old male college student returns from spring break in Fort Lauderdale, Florida, with complaints of acute pain and swelling of the scrotum. Physical examination reveals an exquisitely tender, swollen right testis that is rather hard to examine. The cremasteric reflex is absent, but there is no swelling in the inguinal area. The rest of his genitourinary examination appears to be normal. A urine dip is negative for red and white blood cells. Which of the following is the appropriate next step in management?

d. Ultrasound of the scrotum The patient in the question may have a torsion of his testis that requires immediate attention. Another possibility would be epididymitis, especially if there is a possible antecedent history of sexual activity or urinary tract infection. Prehn sign, although not totally reliable, is elicited by gently lifting the scrotum toward the symphysis. Relief of the pain points to epididymitis; its worsening, to torsion. Doppler ultrasound (or surgical consultation) is a logical first step in this man's evaluation, demonstrating absence of flow in torsion and increased flow in epididymitis.

66. A 3-day-old infant who was found at birth to have anal atresia also has vertebral defects, a VSD, tracheoesophageal fistula, absent left kidney, and shortened arms.

d. VATER

Questions 93 to 95 93. A 6-month-old child has a loud systolic murmur at the left lower sternal border

d. Ventricular septal defect (VSD)

100. Breast-fed infant What supplement is indicated for baby?

d. Vitamin D

274. An exclusively breast-fed 2-year-old is brought to the emergency center with pain in his right leg after a fall. Physical examination reveals a small child with a 3-cm anterior fontanelle, a flattened occiput, a prominent forehead, significant dental caries, bumpy ribs, and bowed extremities. Radiographs reveal a greenstick fracture at the site of pain, along with fraying at the distal ends of the femur.

d. Vitamin D

24. A child can walk well holding on to furniture but is slightly wobbly when walking alone. She uses a neat pincer grasp to pick up a pellet, and she can release a cube into a cup after it has been demonstrated to her. She tries to build a tower of two cubes with variable success. She is most likely at which of the following age?

e. 1 year

82. Vitamin D

e. A 2-year-old presents with nausea, vomiting, poor feeding, abdominal pain, and constipation. On electrocardiogram (ECG) he has decreased Q-T interval. He has calcifications in his kidneys noted on a CT scan done for his abdominal pain

12. A 4-month-old baby boy arrives to the ER cold and stiff. The parents report that he had been healthy and that they put him to bed as usual for the night at the regular time. When they next saw him, in the morning, he was dead. Physical examination is uninformative. A film from a routine skeletal survey is shown below. Which of the following is the most likely diagnosis?

e. Abuse

314. A previously healthy 7-year-old child suddenly complains of a headache and falls to the floor. When examined in the emergency room (ER), he is lethargic and has a left central facial weakness and left hemiparesis with conjugate ocular deviation to the right. Which of the following is the most likely diagnosis?

e. Acute infantile hemiplegia The abrupt onset of a hemisyndrome, especially with the eyes looking away from the paralyzed side, strongly indicates a diagnosis of acute infantile hemiplegia. Most frequently, this represents a *thromboembolic* occlusion of the middle cerebral artery or one of its major branches. The diagnosis has also been used to describe an acute syndrome of fever and partial seizure with resulting hemiparesis. Childhood stroke can result from trauma, infection, a hyper-coagulable state, arteritis, and congenital structural or metabolic disorders.

111. A mother calls you frantic because she has just been diagnosed with chicken pox. She delivered 7 days ago a term infant that appears to be eating, stooling, and urinating well. The child has been afebrile and seems to be doing well. Which of the following is the most appropriate step in management?

e. Advise the mother to continue regular well-baby care for the infant. If a normal full-term newborn is exposed to chicken pox 2 or more days postnatally, VZIG and isolation are not necessary because these babies appear to be at no greater risk for complications than older children. Acyclovir may be used in infants at risk for severe varicella, such as those infants exposed perinatally.

209. A 7-year-old child is brought by his mother for a school physical. His growth parameters show his height to be 50th percentile and his weight to be significantly higher than 95th percentile. His mother complains that he always seems sleepy during the day and that he has started complaining of headaches. His second-grade teacher has commented that he has difficulty staying awake in class. His mother complains that he wakes up the whole house with his snoring at night. Which of the following is the most appropriate next step in evaluating and managing this condition?

e. Arrange for polysomnography Obstructive sleep apnea (OSA) is fairly common, occuring in about 2% of normal children. Factors such as obesity and craniofacial abnormalities increase the risk. Symptoms suggestive of sleep apnea include snoring, frequent nighttime awakenings, changes in behavior, and of course witnessed apnea. While obesity contributes to OSA, poor growth may be a symptom. There is significant overlap with the symptoms of OSA and attention deficit hyperactivity disorder (ADHD), but the reported snoring and sleepiness leans toward OSA. Tonsillectomy and adenoidectomy are frequently performed after children are diagnosed with OSA.

1. Two weeks after a viral syndrome, a 9-year-old boy presents to your clinic with a complaint of several days of weakness of his mouth. In addition to the drooping of the left side of his mouth, you note that he is unable to completely shut his left eye. His smile is asymmetric, but his examination is otherwise normal. Which of the following is the most likely diagnosis?

e. Bell palsy

322. A 6-year-old boy is seen in the office for evaluation of polyuria. Further questioning reveals several months of headache with occasional emesis. Your physical examination reveals a child who is less than 5% for weight. He has mild papilledema. His glucose is normal, and his first urine void specific gravity after a night without liquids is 1.005 g/mL. Which of the following might also be expected to be seen in this patient? What do they have?

e. Bitemporal hemianopsia The findings of poor growth, diabetes insipidus, and papilledema could be explained by a craniopharyngioma. This tumor is one of the most common supratentorial tumors in children, often causing growth failure through disruption of pituitary excretions such as growth hormone.

14. A 2-year-old child presents to the office with a paternal complaint of "bowlegs." The girl has always had bowlegs; her previous pediatrician told the family she would grow out of it. Now, however, it seems to be worsening. Her weight is greater than 95% for age, and she has significant bowing out of her legs and internal tibial torsion; otherwise, her examination is normal. A radiograph of her lower leg is shown. Which of the following is the most likely diagnosis?

e. Blount disease

15. A very concerned mother brings a 2-year-old child to your office because of two episodes of a brief, shrill cry followed by a prolonged expiration and apnea. You have been following this child in your practice since birth and know the child to be a product of a normal pregnancy and delivery, to be growing and developing normally, and to have no chronic medical problems. The first episode occurred immediately after the mother refused to give the child some juice; the child became cyanotic, unconscious, and had generalized clonic jerks. A few moments later the child awakened and had no residual effects. The most recent episode (identical in nature) occurred at the grocery store when the child's father refused to purchase a toy for her. Your physical examination reveals a delightful child without unexpected physical examination findings. Which of the following is the most likely diagnosis?

e. Breath-holding spell

162. An infant of uncertain but seemingly term dates is born via emergent cesarean section for nonreassuring heart tones; the obstetrician has noted little or no amniotic fluid. The infant is small, has abnormally shaped limbs, and an unusual facies. The child has immediate respiratory distress. A chest radiograph reveals a poorly developed chest with little lung tissue.

e. Primary pulmonary hypoplasia Pulmonary hypoplasia due to oligohydramnios (Potter sequence) includes a dysmorphic child (widely spaced eyes, low set ears, broad nose, receding chin, limb abnormalities) and bilateral renal agenesis. These infants have immediate respiratory distress; the condition is not compatible with life.

207. You are asked to evaluate a 4-year-old boy admitted to your local children's hospital with a diagnosis of pneumonia. The parents state that the child has had multiple, intermittent episodes of fever and respiratory difficulty over the past 2 years, including cyanosis, wheezing, and dyspnea; each episode lasts for about 3 days. During each event he has a small amount of hemoptysis, is diagnosed with left lower lobe pneumonia, and improves upon treatment. Repeat radiographs done several days after each event are reportedly normal. His examination on the current admission is significant for findings similar to those described above, as well as digital clubbing. Which of the following is the most appropriate primary recommendation?

e. Bronchoalveolar lavage This child likely has idiopathic pulmonary hemosiderosis (IPH) Bronchoalveolar lavage will reveal hemosiderin-laden macrophages and would be most likely to make the diagnosis. A distinct subset of patients with pulmonary hemosiderosis has hypersensitivity to cow's milk (the association is called Heiner syndrome) and may improve with a diet free of cow's milk products.

107. A 2-year-old girl has had two episodes of Neisseria meningitidis septicemia and is now admitted for Streptococcus pneumoniae septicemia most appropriate diagnostic laboratory test

e. CBC demonstrating Howell-Jolly bodies Asplenia results in Howell-Jolly bodies and also an increased risk for encapsulated organisms such as pneumococcus or meningococcus; a CBC with a peripheral smear can rule out this disease.

494. A 15-year-old male had a painless papule on his genitals that resolved, but he has now developed a unilateral draining inguinal lymphadenitis

e. Chlamydia culture

157. A week-old infant presents with fever and focal seizure.

e. Congenital herpes simplex virus Transmission of HSV from mother to newborn can happen in utero, intrapartum, and postnatally. Intrapartum transmission is most common. Infants born vaginally to a mother with a primary genital herpes infection are at highest risk for disease, with up to a 50% possibility of perinatal transmission.. Infants can display isolated CNS involvement, isolated cutaneous infection, or systemic generalized infection. Treatment usually is with acyclovir; even with therapy, morbidity is high in infants with CNS involvement.

370. Two weeks ago, a 5-year-old boy developed diarrhea, which has persisted to the present time despite dietary management. His stools have been watery, pale, and frothy. He has been afebrile. Microscopic examination of his stools is likely to show which of the following?

e. Cryptosporidium Cryptosporidium has become an important cause of diarrhea in immunocompromised patients, particularly those with AIDS who are not on the highly active antiretroviral therapy (HAART). It can also affect patients who are immunocompetent, and has been recognized as an agent responsible for epidemics of diarrhea in day-care centers.

346. A 10-month-old infant on long-term aspirin therapy for Kawasaki disease develops sudden onset of high fever, chills, diarrhea, and irritability. A rapid swab in your office identifies influenza A, adding her to the long list of influenza patients you have seen this December. Over the next few days, she slowly improves and becomes afebrile. However, 5 days after your last encounter you hear from the hospital that she has presented to the emergency center obtunded and posturing with evidence of liver dysfunction. Which of the following statements about her current condition is correct?

e. Death is usually associated with increased intracranial pressures and herniation. Reye syndrome is an acquired mitochondrial hepatopathy that results from the interaction of an influenza (or varicella) infection and aspirin use. While prevalence has decreased over the last few decades and it is now a rare disease, mortality remains the same at more than 40% of cases. Liver enzymes and ammonia are elevated, but total bilirubin is not. Patients initially present toward the end of a viral infection with sleepiness, emesis, and abnormal liver functions. As the disease progresses, the patient may develop seizures, coma, hyperventilation, and decorticate posturing. Ultimately they may develop respiratory arrest, loss of deep tendon reflexes (DTRs), and fixed and dilated pupils. Death is usually from cerebral edema and subsequent herniation. While aspirin is no longer routinely used in children as an antipyretic or pain reliever, the increase in the use of aspirin in adults with heart disease requires specific counseling for parents of children with influenza and varicella to avoid aspirin use.

419. An otherwise healthy 7-year-old girl is brought to your office by her father because she has some acne, breast development, and fine pubic hair. Which of the following is the most likely etiology for her condition?

e. Early onset of "normal" puberty (constitutional) The patient in the question appears to have "true sexual precocity," implying that the gonads have matured in response to the secretion of pituitary gonadotropins and have begun secreting sex steroids, causing the development of secondary sexual characteristics. Thus, ovarian tumors and exogenous estrogens, which suppress the function of the pituitary gland, do not cause true precocious puberty; rather, they cause isolated premature telarche and/or vaginal bleeding without pubic hair, body odor, and acne. In girls, the most common form of true precocious puberty is idiopathic and is thought to be caused by early maturation of an otherwise normal hypothalamic-pituitarygonadal feedback system. In boys, true precocious puberty is relatively rare and is more likely to be caused by lesions of the CNS.

Questions 43 to 48 43. A 1-week-old child's mother complains that the child has a transient rash that has splotchy areas of erythema with a central clear pustule. Your microscopic examination of the liquid in the pustule reveals eosinophils.

e. Erythema toxicum

398. A 4-year-old previously well African American boy is brought to the office by his aunt. She reports that he developed pallor, dark urine, and jaundice over the past few days. He is taking trimethoprim sulfamethoxazole for otitis media. The CBC in the office shows a low hemoglobin and hematocrit, while his "stat" serum electrolytes, blood urea nitrogen (BUN), and chemistries are remarkable only for an elevation of his bilirubin levels. His aunt seems to recall his 8-year-old brother having had an "allergic reaction" to aspirin, which also caused a short-lived period of anemia and jaundice. Which of the following is the most likely cause of this patient's symptoms?

e. Glucose-6-phosphate dehydrogenase deficiency Synthesis of the RBC enzyme glucose-6-phosphate dehydrogenase (G6PD) is determined by genes on the X chromosome, and the pattern of inheritance is X-linked recessive. The disease occurs, though less commonly, in other ethnic groups, including Middle Eastern, African, and Asian groups. Deficiency of G6PD compromises the generation of reduced glutathione and upon exposure to oxidant agents. The older RBCs containing Heinz bodies (insoluble precipitates resulting from oxidation), the "bite cells" (RBCs after the removal of the Heinz bodies), and cell fragments are removed from the circulation within 3 to 4 days.

292. A 4-year-old boy and his family have recently visited a local amusement park. Several of the family members developed "gastroenteritis" with fever and diarrhea, but the 4-year-old's stool was slightly different, as it contained blood. His mother reports that in the past 24 hours he developed pallor and lethargy; she relates that his face looks swollen and that he has been urinating very little. Laboratory evaluation reveals a hematocrit of 28% and a platelet count of 72,000/μL. He has blood and protein in the urine. Which of the following diagnoses is most likely to explain these symptoms?

e. Hemolytic-uremic syndrome HUS is most common in children younger than 4 years of age and is characterized by an acute microangiopathic hemolytic anemia, thrombocytopenia from increased platelet utilization, and renal insufficiency from vascular endothelial injury and local fibrin deposition. Laboratory findings associated with hemolytic uremic syndrome include low hemoglobin level, decreased platelet count, hypoalbuminemia, and evidence of hemolysis on peripheral smear (burr cells, helmet cells, schistocytes). Infection by the verotoxin- producing E coli 0157:H7 has been implicated as a cause of HUS.

450. A 6-year-old child who complains of numbness and tingling of the hands, and later develops tonic-clonic seizures. What are the PO4 and Ca levels in this patient?

e. High PO4, low Ca Hypoparathyroidism is unusual in children outside the newborn period, usually presenting with neuromuscular instability such as seizures, with numbness and tingling possibly preceding the seizures, like those in the patient in the question, who had numbness and tingling of the hands and who later developed seizures. In response to low concentrations of parathyroid hormone, there is reduced bone resorption. In the kidney, there is reduced excretion of phosphate and reduced formation of 1,25- dihy-droxyvitamin D. The reduced 1,25-dihydroxyvitamin D formation in turn reduces the absorption of calcium and, secondarily, of phosphorus from the gut. The net effect is hypocalcemia and hyperphosphatemia.

448. A short, 4-year-old, mentally retarded child with brachydactyly of the fourth and fifth digits; obesity with round facies; short neck; subcapsular cataracts; and cutaneous, subcutaneous, and perivascular calcifications of the basal ganglia What are the PO4 and Ca levels in this patient?

e. High PO4, low Ca Patients with pseudohypoparathyroidism have the same chemical abnormality (low Ca, high PO4) as those with hypoparathyroidism. They are distinguished from the latter group by the phenotypic features demonstrated by the girl in the question (Albright hereditary osteodystrophy) and high serum concentration of parathyroid hormone. The basic abnormality in these patients is the unresponsiveness of the renal tubules to parathyroid hormone. These patients, if given parathyroid hormone, have increased urinary excretion of cyclic AMP but not of phosphate.

57. A 14-year-old on a mountain-climbing expedition in December becomes tired, clumsy, and begins to hallucinate. His heart rate is 45 beats per minute

e. Hypothermia

373. A patient presents to the emergency center with a 6-hour history of fever to 38.9°C (102°F). Her mother reports that the patient appeared to be feeling poorly, that she had been eating less than normal, and that she vomited once. About 2 hours prior to arrival at the ER, the mother states that she noted a few purple spots scattered about the body on the patient, especially on the buttocks and legs. On the 30-minute ride to the ER, the purple areas spread rapidly and became coalesced in areas, and the patient is now obtunded. Which of the following is the most likely diagnosis?

e. Meningococcemia

Questions 275 to 277 275. A 3-week-old child currently admitted to the hospital for pneumonia who gags and chokes during feedings

e. Modified barium swallow (aka video fluoroscopic swallowing exam) An infant who gags and chokes while feeding may have an uncoordinated suck-swallow reflex, or may have significant gastroesophageal reflux. More rarely there may be an H-type tracheoesophageal fistula. A modified barium swallow with fluoroscopy allows direct visualization of the swallow reflex. During the modified barium swallow, the patient may be given different consistencies of food to document if thickened feeds improve swallowing mechanics. A normal barium study does not, however, rule out a fistula.

459. An 11-year-old boy with central diabetes insipidus secondary to an automobile accident

e. Na+ 155, K+ 5.5 In the absence of vasopressin (central diabetes insipidus), renal collecting tubules are impermeable to water, resulting in the excretion of hypotonic urine. Patients with diabetes insipidus present with polyuria and polydipsia. Net loss of water leads to dehydration and hemoconcentration and, therefore, to relatively high serum concentrations of sodium and potassium.

460. A 2-year-old girl with nephrogenic diabetes insipidus

e. Na+ 155, K+ 5.5 Nephrogenic diabetes insipidus presents similarly to central diabetes insipidus. Patients with diabetes insipidus present with polyuria and polydipsia. Net loss of water leads to dehydration and hemoconcentration and, therefore, to relatively high serum concentrations of sodium and potassium.

185. A newborn infant has mild cyanosis, diaphoresis, poor peripheral pulses, hepatomegaly, and cardiomegaly. Respiratory rate is 60 breaths per minute, and heart rate is 250 beats per minute. The child most likely has congestive heart failure caused by which of the following?

e. Paroxysmal atrial tachycardia

353. A 15-month-old boy is brought to the ER because of fever and a rash. Six hours earlier he was fine, except for tugging on his ears; another physician diagnosed otitis media and prescribed amoxicillin. During the interim period, the child has developed an erythematous rash on his face, trunk, and extremities. Some of the lesions, which are of variable size, do not blanch on pressure. The child is now very irritable, and he does not interact well with the examiner. Temperature is 39.5°C (103.1°F). He continues to have injected, immobile tympanic membranes, but you are concerned about his change in mental status. Which of the following is the most appropriate next step in the management of this infant?

e. Perform a lumbar puncture Unsuspected bacteremia caused by H influenzae type B (now rare), Neisseria meningitidis, or S pneumoniae (decreasing in frequency secondary to vaccination) should be considered before prescribing treatment for otitis media in a young, febrile, toxic-appearing infant. In this situation, blood culture should be performed before antibiotic therapy is initiated, and examination of the CSF is indicated if meningitis is suspected.

27. A 1-year-old presents for a well-child checkup, but the parents are concerned about giving the child his immunizations. Which of the following is a true contraindication to the administration of the fourth DTaP (diphtheria and tetanus toxoid and acellular pertussis) vaccine?

e. Prolonged seizures 6 days after the last DTaP vaccine

313. A 14-year-old girl with a history of seizures is admitted to the hospital with the diagnosis of status epilepticus. Her valproic acid level is in the therapeutic range. You arrange a 24-hour video electroencephalogram (EEG). During the EEG, she has several episodes of tonic and clonic movements with moaning and crying, with no loss of bowel or bladder control. The neurologist tells you that during the events the EEG had excessive muscle artifact but no epileptiform discharges. Which of the following treatments is the most appropriate for this condition?

e. Request a psychiatric evaluation Seizure- like activity with no epileptiform activity on EEG is consistent with pseudoseizure. These episodes may be very convincing for seizures and may include unusual posturing and sounds, but typically do not involve loss of bowel or bladder control. These episodes can be deliberate or part of a conversion disorder, and do not require treatment with antiepileptic medications. However, many patients with pseudoseizures also have true epileptic seizures, so withdrawal of antiepileptic medication would be inappropriate at this point.

367. An 8-year-old Cub Scout who returned from an outing 9 days ago is brought to the clinic with the rapid onset of fever, headache, muscle pain, and rash. The maculopapular rash began on the flexor surfaces of the wrist and has become petechial as it spread inward to his trunk. Which of the following is the most likely diagnosis?

e. Rocky Mountain spotted fever

347. An 18-month-old child presents to the emergency center having had a brief, generalized tonic-clonic seizure. He is now postictal and has a temperature of 40°C (104°F). During the lumbar puncture (which ultimately proves to be normal), he has a large, watery stool that has both blood and mucus in it. Which of the following is the most likely diagnosis in this patient?

e. Shigella Clinical manifestations of shigellosis range from watery stools for several days to severe infection with high fever, abdominal pain, and generalized seizures.

74. A 4-year-old girl comes into the ER after eating a bottleful of small, chewable pills she found while at her grandfather's house. She has an increased respiratory rate, elevated temperature, vomiting, and is disoriented. She is intermittently complaining that "a bell is ringing" in her ears. She has a metabolic acidosis on an arterial blood gas.

e. Sodium bicarbonate Salicylate poisoning produces metabolic acidosis and respiratory alkalosis (although this latter feature is often missed in young children), hyperglycemia or hypoglycemia, paradoxical aciduria, dehydration, and lethargy. Excretion of salicylates in the urine can be markedly enhanced by the administration of acetazolamide and IV sodium bicarbonate. Hemodialysis can also be used.

51. Parents hear over their baby monitor that their 5-year-old girl regularly calls out during the night. When the parents check on her, she is sleeping comfortably and is in no apparent distress.

e. Somniloquy

Questions 172 to 174 172. A 1-day-old infant has a fronto-occipital head circumference that is 2 cm larger than the initial measurement done several hours before, the scalp has a "squishy" feel to it, and the infant has developed tachycardia.

e. Subgaleal hemorrhage

35. A 14-year-old high school student arrives to your clinic for well-child care. In reviewing his records you determine that his most recent immunization for tetanus was at 4 years of age. Which of the following should you recommend?

e. Tetanus toxoid, reduced diphtheria toxoid, and acellular pertussis vaccine adsorbed (Tdap)

70. A 3-year-old refuses to walk, is afebrile, had an upper respiratory tract infection a week ago, has right hip pain with movement, and has a normal WBC count.

e. Transient synovitis

372. An 8-year-old immigrant from rural Central America presents with complaints of weakness, facial swelling, muscle pain, and fever. A CBC reveals marked eosinophilia. Which of the following parasites is most likely to be responsible? How to treat?

e. Trichinella spiralis One to seven days following the ingestion of pork or other improperly cooked meat infected with T spiralis, symptoms develop, including abdominal pain, nausea, vomiting, and malaise. During the second week, muscle invasion occurs, which causes edema of eyelids, myalgia, weakness, fever, and eosinophilia. The muscle organisms can become encysted and remain viable for years. Therapy is with mebendazole or albendazole.

473. A previously normal father of one of your patients, from a family with members known to have fragile X syndrome, develops ataxia and tremor

e. Triplet repeat expansion disorder

441. An 18-year-old girl has hepatosplenomegaly, an intention tremor, dysarthria, dystonia, and deterioration in her school performance. She also developed abnormal urine with excess glucose, protein, and uric acid. She has a several-year history of elevated liver enzymes of unknown etiology. Which of the following best explains her condition?

e. Wilson disease Wilson disease is an autosomal recessive disorder characterized by liver disease (usually seen in childhood), neurologic and behavioral disturbances (seen by adolescence), renal tubular dysfunction (Fanconi syndrome), and eye findings (Kayser- Fleischer rings). Its multisystem manifestations are caused by the deposition of copper in various tissues (resulting in low serum levels), and therapy is aimed at the prevention of accumulation of copper. Defective metabolism of the copper-binding protein ceruloplasmin (usually reduced) has been demonstrated by some.

97. Administration of phenytoin What supplement is indicated for baby?

f. Folate

101. Sickle-cell disease What supplement is indicated for baby?

f. Folate Patients with hemolytic anemia, such as sickle-cell disease, have an ongoing compensatory erythropoiesis. To supply the increased need of rapidly dividing RBC precursors for folate, supplementation is necessary.

Questions 303 to 306 303. An 8-year-old boy with the intermittent complaint of "burning" when he urinates and who has trace blood on his urine dip test.

f. Idiopathic hypercalciuria Idiopathic hypercalciuria causes recurrent gross hematuria, persistent microscopic hematuria, and complaints of dysuria or abdominal pain without initial stone formation. Over time, however, stones may form in 15% of cases.

270. A 9-month-old infant, who has been fed cow's milk exclusively for 4 months, is tachycardic and pale.

f. Iron

267. A 7-year-old has been vomiting for 2 days and has had diarrhea for 1 day. He now notes that he has small streaks of blood in his emesis. The rest of his family has had similar symptoms

f. No immediate intervention

104. A 5-month-old infant is admitted with severe varicella infection. The lesions cover the infant's entire body, and the infant is beginning to show symptoms of respiratory distress. Past medical history is significant for a history of atopic dermatitis. The family also notes frequent epistaxis; the last episode required nasal packing in the ED. most appropriate diagnostic laboratory test

f. Platelet count Wiskott-Aldrich syndrome must be considered in a patient with severe eczema and unusual infections, and is a strong possibility with this history if the platelet count is low (but is unlikely if the platelet count is normal). Other findings include eosinophilia and elevated IgE.

161. A postterm infant is born at home after a prolonged and difficult labor. The maternal grandmother brings the infant to the hospital at 1 hour of life because of fast breathing. Grandmother notes that the child seemed well for a while, but then developed increased work of breathing. Physical examination reveals an infant in moderate respiratory distress with diminished breath sounds on the left. Chest radiograph reveals the heart to be pushed to the right side and loss of lung markings in the left lung field.

f. Pneumothorax

86. A woman with long-standing hypertension treated with angiotensinconverting enzyme (ACE) inhibitors

f. Renal dysgenesis Angiotensin-converting enzyme (ACE) antihypertensives have been associated with renal dysgenesis, oligohydramnios, and skull ossification defects.

48. A newborn's father complains that his son has dandruff, with many waxy flakes of skin on the scalp. When he scrapes the lesions, hair often comes off with the flakes of skin. In addition, the baby has flaking of the eyebrows.

f. Seborrheic dermatitis

90. A 2-week-old boy is brought by his mother to the clinic; he has scaly, yellow patches on his scalp with associated hair loss.

f. Topical steroids or a selenium sulfide-containing product Treatment for common seborrheic dermatitis consists of antiseborrheic shampoos; topical corticosteroids may be used for inflamed lesions.

Questions 59 to 62 59. Megaloblastic anemia, growth failure, paresthesias, sensory defects, developmental regression, weakness, and fatigue

f. Vitamin B12 deficiency

80. Nicotinic acid

g. After getting into his mother's bottle of vitamins, a 3-year-old has burning, tingling, and itching on his arms as well as a reddened face, arms, and chest.

87. A primiparous mother late in her first trimester has a fever and "3-day" measles.

g. Cataracts Congenital rubella syndrome

Questions 464 to 470 464. A 10-year-old boy with hypermobile joints and poor wound healing

g. Ehlers-Danlos syndrome

47. A newborn's mother complains that her infant seems to have very small white dots all over his nose. The dots do not wipe off with bathing, but they are also not erythematous

g. Milia

391. A 2-month-old boy with a 3-day history of upper respiratory infection who suddenly develops high fever, cough, and respiratory distress; within 48 hours, the patient has developed a pneumatocele and a left-sided pneumothorax.

g. Staphylococcal pneumonia Staphylococcal pneumonia is caused by S aureus. It is a *rapidly progressive and life-threatening* form of pneumonia most commonly seen in children less than 1 year of age. Commonly, the child has a URI for several days, with the abrupt onset of fever and respiratory distress. *Pleural effusion, empyema, and pyopneumothorax are common complications*. Laboratory evidence of this disease can include a markedly elevated WBC count with left shift. Radiographic findings include nonspecific bronchopneumonia early in the disease, which later becomes more dense and homogeneous and involves an entire lobe or hemithorax.

160. A large-for-gestation-age term infant is delivered via scheduled cesarean section develops, at 15 minutes of age, tachypnea, grunting, flaring, and retractions. The child does not move his left arm well, but you find no clavicular fracture. A chest radiograph shows the left diaphragm to be markedly higher than the right.

i. Phrenic nerve paralysis Infants with upper brachial plexus injury (cervical nerves 3, 4, 5) can also have ipsilateral phrenic nerve paralysis. These infants can present with labored, irregular breathings and cyanosis; the injury is usually unilateral. Confirmation of the diagnosis is made with ultrasound or fluoroscopy, which confirms "seesaw" movements of the diaphragm during respiration.

60. Photophobia, blurred vision, burning and itching of eyes, poor growth, cheilosis

i. Riboflavin deficiency

177. You are seeing a 2-year-old boy for the first time. His father denies any past medical or surgical history, but does note that the child's day care recently sent a note home asking about several episodes, usually after the child does not get what he wants, when he "breathes funny" and sits in a corner with his knees under his chin for a few minutes. The day-care staffers think this "self-imposed time-out" is a good thing, but they worry about the breathing. One teacher even thought he once looked blue, but decided that it was probably because of the finger paints he had been using. On examination, you identify a right ventricular impulse, a systolic thrill along the left sternal border, and a harsh systolic murmur (loudest at the left sternal border but radiating through the lung fields). His chest radiograph and ECG are shown. Which of the following congenital cardiac lesions would you expect to find in this child?

אין תשובה שלך ואין תשובות של הספר


Conjuntos de estudio relacionados

Unit 12 Lesson 58 Notgrass Exploring America

View Set

English File pre-intermediate Unit 1 - 1

View Set

Chapter 7: Introduction to Structured Query Language (SQL)

View Set

Midterm review for CIS 263 (ethical hacking and countermeasures)

View Set

MGT 3050 Exam One Practice Questions

View Set